Крок 1 - Стоматологія 2015 (буклет)

1 / 200
При цитологічних дослідженнях було виявлено велику кількість різних молекул т-РНК, які доставляють амінокислоти до рибосоми. Кількість рїзних типів т-РНК у клітині буде дорівнювати кількості: Cytological studies have revealed a large number of different t-RNA molecules that deliver amino acids to the ribosome. The number of different types of t-RNA in the cell will be equal to:

Триплетів, що кодують амінокислоти Triplets encoding amino acids

Нуклеотидїв Nucleotides

Білків, синтезованих у клітині Proteins synthesized in the cell

Різних типів і-РНК Different types of i-RNA

Амінокислот Amino acid

2 / 200
У районах Південної Африки у людей розповсюджена серпоподібноклїтинна анемія, при якій еритроцити мають форму серпа внаслідок зміни в молекулі гемоглобіну амінокислоти глутамату на валін. Причиною цієї хвороби є: In areas of South Africa, people have sickle cell anemia, in which red blood cells have a sickle shape due to a change in the hemoglobin molecule from the amino acid glutamate to valine. The cause of this disease is:

Геномна мутація Genomic mutation

Кросинговер Crossover

Генна мутація Gene mutation

Трансдукція Transduction

Порушення механізмів реалізації генетичної інформації Violation of genetic information implementation mechanisms

3 / 200
При обстеженні дівчини 18-ти років знайдені наступні ознаки: недорозвинення яєчників, широкі плечі, вузький таз, вкорочення нижніх кінцівок, 'шия сфінкса', розумовий розвиток не порушений. Встановлено діагноз: синдром Шерешевського-Тернера. Яке хромосомне порушення у хворої? During the examination of an 18-year-old girl, the following signs were found: underdevelopment of the ovaries, broad shoulders, narrow pelvis, shortening of the lower limbs, 'neck of the sphinx', mental development is not impaired The diagnosis was established: Shereshevsky-Turner syndrome. What chromosomal disorder does the patient have?

Трисомїя 13 Trisomy 13

Трисомїя Х Trisomy X

Моносомїя Х Monosomy X

Трисомїя 18 Trisomy 18

Нульсомїя Х Nulsomia X

4 / 200
У хворого виявлена короткочасна пневмонія. Міграція личинок якого гельмінта може призвести до цієї хвороби? The patient was diagnosed with short-term pneumonia. The migration of larvae of which helminth can lead to this disease?

Аскарида Ascarida

Альвеокок Alveococcus

Карликовий ціп’як Dwarf creeper

Волосоголовець Hairhead

Гострик Hostrik

5 / 200
При огляді хворого з ранами, що кровоточать, лікар виявив пошкодження тканин личинками, а також локальні місця нагноєння. Діагноз: облігатний мїаз. Збудником цього захворювання є: When examining a patient with bleeding wounds, the doctor found tissue damage by larvae, as well as local suppuration. Diagnosis: obligate myiasis. The causative agent of this disease is:

Триатомовий клоп Triatomic bug

Муха хатня Housefly

Муха вольфартовая Wolfarth fly

Жигалка осіння Autumn lighter

Муха цеце Tsetse fly

6 / 200
Дівчині 18-ти років встановлено попередній діагноз - синдром Шерешевського-Тернера. Це можна підтвердити за допомогою такого метода: An 18-year-old girl has a preliminary diagnosis of Shereshevsky-Turner syndrome. This can be confirmed using the following method:

Близнюковий Twin

Генеалогічний Genealogical

Біохімічний Biochemical

Цитогенетичний Cytogenetic

Дерматогліфіка Dermatoglyphics

7 / 200
У хворого з підозрою на одне з протозойних захворювань досліджено пунктат лімфатичного вузла. В препараті, забарвленому за Романовським-Гімзою, виявлено тільця півмісяцевої форми із загостреним кінцем, блакитною цитоплазмою, ядром червоного кольору. Яких найпростіших виявлено в мазках? A patient suspected of having one of the protozoan diseases had a lymph node punctate examined. In the preparation, stained according to Romanovsky-Giemsa, crescent-shaped bodies with a pointed end, blue cytoplasm were found , with a red core. What protozoa were found in the smears?

Токсоплазми Toxoplasma

Трипаносоми Trypanosomes

Дерматотропні лейшманії Dermatotropic leishmania

Вісцеротропні лейшманії Viscerotropic leishmania

Малярійні плазмодії Malarial plasmodia

8 / 200
Після черепно-мозкової травми у хворого 38-ми років відсутнє відчуття нюху і смаку. Які коркові центри кінцевого мозку постраждали? After a craniocerebral injury, a 38-year-old patient has no sense of smell and taste. What cortical centers of the terminal brain were affected?

Gurus postcentralis Gurus postcentralis

Gyrus temporalis superior Gyrus temporalis superior

Uncus Uncus

Operculum frontale Operculum frontale

Sul. calcarinus Sul. calcarinus

9 / 200
Оглядаючи ротову порожнину у хворого 19-ти років, стоматолог звернув увагу на зруйнований язиково-дистальний горбик жувальної поверхні лівого першого верхнього моляра. Цей горбик називається: While examining the oral cavity of a 19-year-old patient, the dentist noticed a destroyed lingual-distal tubercle of the chewing surface of the left first upper molar. This tubercle is called:

Мезоконус Mesocone

Гіпоконус Hypoconus

Метаконус Metacone

Протоконус Protocone

Параконус Paracone

10 / 200
Є потреба катетеризації сечового міхура у чоловіка. В якій частині уретри або структурі може виникати опір катетеру? There is a need for bladder catheterization in a man. In what part of the urethra or structure can resistance to the catheter occur?

Перетинчаста Membranous

Простатична Prostatic

Зовнішній отвір сечівника External orifice of urethra

Внутрішній отвір сечівника Internal orifice of urethra

Губчаста Spongy

11 / 200
При огляді хворого невропатолог відзначає асиметрію обличчя: згладженість складок шкіри лоба, око повністю не закривається, на одному боці очна щілина ширша, рот перекошений. Який нерв уражений? When examining the patient, the neurologist notes asymmetry of the face: the forehead skin folds are smoothed, the eye does not close completely, the eye slit is wider on one side, the mouth is crooked. Which nerve is affected?

Верхньощелепний Maxillary

Очний Full-time

Нижньощелепний Mandibular

Лицевий Face

Додатковий Additional

12 / 200
У хворого відкритий рот, він не може його закрити і членороздільно говорити. Яке ураження у хворого? The patient's mouth is open, he cannot close it and speak articulately. What is the patient's lesion?

Вивих нижньої щелепи Dislocation of the lower jaw

Перелом нижньої щелепи Fracture of lower jaw

Параліч жувальних м’язів Paralysis of masticatory muscles

Ураження рухового центру мови Language motor center damage

Стрес Stress

13 / 200
Під час операції закриття природженої щілини піднебіння (уранопластики) при збиванні долотом крилоподібного гачка ушкоджено великий піднебінний канал. Виникла кровотеча. Яку артерію ушкоджено? During the operation to close the congenital cleft palate (uranoplasty) when the pterygoid hook was knocked down with a chisel, the large palatine canal was damaged. Bleeding occurred. Which artery was damaged?

Низхідна піднебінна Descending Palate

Задня верхня альвеолярна Posterior superior alveolar

Клиноподібна Wedge

Висхідна піднебінна Eastern Palate

Висхідна глоткова Ascending pharyngeal

14 / 200
У хворого 60-и років виявлено розширення вен стравоходу, прямої кишки та підшкірних вен передньої черевної стінки. Система якої вени ушкоджена? In a 60-year-old patient, dilatation of the veins of the esophagus, rectum, and subcutaneous veins of the anterior abdominal wall was detected. Which vein system is damaged?

Ворітна Voritna

Верхня порожниста вена Superior vena cava

Верхня брижова Upper Mesentery

Нижня порожниста вена Inferior vena cava

Непарна вена Odd vein

15 / 200
У травмованого 27-ми років двосторонній вивих скронево-нижньощелепного суглоба. Які основні зв’язки суглоба будуть ушкоджені? A 27-year-old injured person has a bilateral dislocation of the temporomandibular joint. What major ligaments of the joint will be damaged?

Крилоподібно-щелепні Prytoid-jaw

Клиноподібно-щелепні Square-jaw

Латеральні Laterals

Шило-щелепні Awl-jaw

Внутрішньосуглобові Intra-articular

16 / 200
У хворого внаслідок пошкодження шкіри в середній ділянці грудинно-ключично- соскоподібного м’яза виникла повітряна емболія. Яка вена шиї була травмована? The patient had an air embolism due to damage to the skin in the middle area of the sternocleidomastoid muscle. Which neck vein was injured?

Передня яремна Anterior jugular

Зовнішня яремна External jugular

Задня вушна Back auricle

Поперечна вена шиї Transverse jugular vein

Внутрішня яремна Internal jugular

17 / 200
У хворого виникають болі при жуванні, утруднення при піднятті вверх нижньої щелепи (при закриванні рота). Функція яких м’язів порушена? The patient has pain when chewing, difficulty in raising the lower jaw upwards (when closing the mouth). Which muscles function is impaired?

Скроневий, жувальний, латеральний крилоподібний Temporal, masseter, lateral pterygoid

Латеральний і медіальний крилоподібні, жувальний Lateral and medial pterygoid, masticatory

Скроневий, жувальний, щелепно-під’язиковий Temporal, masticatory, maxillohyoid

Скроневий, жувальний, медіальний крилоподібний Temporal, masseter, medial pterygoid

Скроневий, латеральний і медіальний крилоподібні Temporal, lateral and medial pterygoid

18 / 200
У дитини 2-х років після перенесеного грипу з’явилися скарги на біль у вусі. Лікар виявив зниження слуху та запалення середнього вуха. Яким шляхом інфекція потрапила до середнього вуха? A 2-year-old child complained of ear pain after suffering the flu. The doctor found hearing loss and inflammation of the middle ear. How did the infection get to the middle ears?

Через foramen jugularis Through the foramen jugularis

Через canalis caroticus Through canalis caroticus

Через слухову трубу Through the auditory tube

Через atrium mastoideum Through atrium mastoideum

Через canalis nasolacrimalis Through canalis nasolacrimalis

19 / 200
В гістопрепараті представлена частина органу, що містить багатошаровий плоский незроговілий епітелій та нижче розташовані сполучнотканинні пластинки, які не містять судин. Який це орган? In the histopreparation, a part of an organ is presented, which contains a multi-layered flat non-keratinized epithelium and connective tissue plates that do not contain blood vessels are located below. What kind of organ is this?

Слизова оболонка стравоходу Esophageal mucosa

Кришталик Crystal

Рогівка Cornea

Слизова оболонка ротової порожнини Mucosa of the oral cavity

Сітківка Retina

20 / 200
У препараті червоного кісткового мозку людини визначаються скупчення гігантських клітин, розташованих в тісному контакті з синусоїдними капілярами. Назвіть формені елементи крові, які утворюються з цих клітин: In the preparation of red bone marrow of a person, clusters of giant cells located in close contact with sinusoidal capillaries are determined. Name the formed blood elements that are formed from these cells:

Еритроцити Erythrocytes

Лімфоцити Lymphocytes

Лейкоцити Leukocytes

Кров’яні пластинки Platelets

Моноцити Monocytes

21 / 200
На гістологічному препараті пухкої сполучної тканини знайдено відносно великі клітини, заповнені базофільною метахроматичною зернистістю; гістохімічно встановлено, що гранули містять гепарин та гістамін. Що це за клітини? Relatively large cells filled with basophilic metachromatic granules were found on the histological specimen of loose connective tissue; it was histochemically established that the granules contain heparin and histamine. What kind of cells are these?

Плазмоцити Plasmocytes

Тканинні базофіли (тучні клітини). Tissue basophils (mast cells).

Фібробласти Fibroblasts

Адипоцити Adipocytes

Макрофаги Macrophages

22 / 200
При ендоскопічному дослідженні у хворого з хронічним ентероколітом (запалення кишки) спостерігається відсутність специфічних структур рельєфу тонкої кишки. Які компоненти визначають особливості рельєфу слизової оболонки тонкої кишки в нормі? During endoscopic examination of a patient with chronic enterocolitis (inflammation of the intestine), the absence of specific structures of the relief of the small intestine is observed. What components determine the features of the relief of the mucous membrane of the small intestine in normal?

Поля, складки, ямки Fields, folds, dimples

Гаустри, ворсинки, крипти Houstras, villi, crypts

Поля, ворсинки Fields, hairs

Косо-спіральні складки Oblique spiral folds

Циркулярні складки, ворсинки та крипти Circular folds, villi and crypts

23 / 200
Морфологічні дослідження селезінки виявили активізацію імунних реакцій в організмі. В яких структурах даного органу починається антигензалежна проліферація Т-лімфоцитів? Morphological studies of the spleen revealed the activation of immune reactions in the body. In which structures of this organ does the antigen-dependent proliferation of T-lymphocytes begin?

Мантійна зона білої пульпи White Pulp Mantle Zone

Червона пульпа Red Pulp

Маргінальна зона білої пульпи Marginal zone of white pulp

Центральна зона білої пульпи Central zone of white pulp

Периартеріальна зона білої пульпи Periarterial zone of the white pulp

24 / 200
У препараті в одному з судин мікроциркуляторного русла середня оболонка утворена 1-2 шарами гладеньких міоцитів, які розташовані поодинці і мають спіралеподібний напрямок. Зовнішня оболонка представлена тонким шаром пухкої сполучної тканини. Вкажіть вид судини: In the preparation, in one of the vessels of the microcirculatory bed, the middle shell is formed by 1-2 layers of smooth myocytes, which are located singly and have a spiral direction. The outer shell is represented by a thin layer of loose connective tissue tissue. Specify the type of vessel:

Посткапіляр Postcapillary

Артеріола Arteriole

Венула Venula

Капіляр Capillary

Артеріоловенулярний анастомоз Arteriovenular anastomosis

25 / 200
Шкідливі екологічні чинники призвели до різкого падіння ендоцитозу і екзоцитозу в клітинах печінки та крові. Який шар плазмолеми постраждав найбільше? Harmful environmental factors led to a sharp drop in endocytosis and exocytosis in liver and blood cells. Which layer of the plasmolemma suffered the most?

Надмембранний Supremembrane

Ліпопротеїновий Lipoprotein

Інтегральний Integral

Глікокалікс Glycocalyx

Кортикальний Cortical

26 / 200
У ході утворення плащового дентину у молочному зубі відбулося порушення секреторної активності одонтобластів. Утворення яких волокон зміниться? During the formation of mantle dentin in a milk tooth, the secretory activity of odontoblasts was disturbed. Which fibers will be formed?

Тангенційні колагенові волокна Ебнера Ebner's tangential collagen fibers

Нервові Nervous

Радіальні колагенові волокна Корфа Radial Korf collagen fibers

Еластичні Elastic

Ретикулярні Reticular

27 / 200
При утворенні зародка людини можна спостерігати появу у його складі порожнини, світлих дрібних бластомерів на периферії та темних великих бластомерів на одному з полюсів. Як називається зародок на цій стадії розвитку? During the formation of a human embryo, you can observe the appearance of a cavity, light small blastomeres on the periphery and dark large blastomeres at one of the poles. What is the name of an embryo at this stage of development ?

Зигота Zygote

Гаструла Gastrula

Бластоциста Blastocyst

Зародковий диск Embryonic disk

Морула Morula

28 / 200
При непрямому гістогенезі кісткової тканини трубчастих кісток між епіфізарним та діафізарним центрами окостеніння утворюється пластинка, що в подальшому забезпечує ріст кісток у довжину. Як називається ця структура? In the case of indirect histogenesis of bone tissue of tubular bones, a plate is formed between the epiphyseal and diaphyseal centers of ossification, which subsequently ensures bone growth in length. What is this structure called?

Кісткова пластинка Bone plate

Шар внутрішніх генеральних пластинок Layer of internal general plates

Метафізарна пластинка Metaphyseal plate

Остеон Osteon

Кісткова манжетка Bone Cuff

29 / 200
У хворого з пересадженим серцем при фізичному навантаженні збільшився хвилинний об’єм крові. Який механізм регуляції забезпечує ці зміни? The minute volume of blood increased during physical exertion in a patient with a transplanted heart. What regulation mechanism ensures these changes?

Парасимпатичні умовні рефлекси Parasympathetic conditioned reflexes

Симпатичні безумовні рефлекси Sympathetic unconditioned reflexes

Симпатичні умовні рефлекси Sympathetic conditioned reflexes

Катехоламіни Catecholamines

Парасимпатичні безумовні рефлекси Parasympathetic unconditioned reflexes

30 / 200
Хворому з гіперсекрецією шлункового соку лікар рекомендував виключити з харчового раціону: A patient with hypersecretion of gastric juice was recommended by the doctor to exclude from the diet:

Солодке Sweet

Молоко Milk

М’ясні бульйони Meat broths

Солоне Salty

Білий хліб White bread

31 / 200
Перед проведенням оперативного втручання з’ясовано, що у людини час кровотечі збільшений до 15 хвилин. Дефіцит яких формених елементів у складі крові може бути причиною таких змін? Before the surgical intervention, it was found that the person's bleeding time was increased to 15 minutes. A deficiency of what formed elements in the composition of the blood could be the cause of such changes?

Тромбоцити Platelets

Лімфоцити Lymphocytes

Лейкоцити Leukocytes

Моноцити Monocytes

Еритроцити Erythrocytes

32 / 200
Досліджуються рецептори, інформація від яких прямує до кори головного мозку без участі таламусу. Які це рецептори? Receptors from which information is sent to the cerebral cortex without the involvement of the thalamus are being studied. What are these receptors?

Нюхові Olfactory

Дотикові Touch

Смакові Tasty

Зорові Visual

Слухові Aural

33 / 200
При дослідженні ізольованого кардіоміоциту встановлено, що він НЕ ГЕНЕРУЄ імпульси збудження автоматично. З якої структури серця отриманий кардіоміоцит? When examining an isolated cardiomyocyte, it was found that it does NOT generate excitation pulses automatically. From which heart structure is the cardiomyocyte derived?

Шлуночок Venticle

Сино-атріальний вузол Sino-atrial node

Атріовентрикулярний вузол Atrioventricular node

Пучок Гіса His Bundle

Волокно Пуркін’є Purkinje fiber

34 / 200
Після руйнування структур ЦНС тварина втратила орієнтувальні рефлекси. Що саме зруйнували? After the destruction of the structures of the central nervous system, the animal lost orientation reflexes. What exactly was destroyed?

Медіальні ретикулярні ядра Medial reticular nuclei

Чорна речовина Black Matter

Червоні ядра Red kernels

Латеральні вестибулярні ядра Lateral vestibular nuclei

Чотиригорбкове тіло Four humped body

35 / 200
При визначенні енерговитрат організму людини встановлено, що дихальний коефіцієнт дорівнює 1,0. Це означає, що у клітинах досліджуваного переважно окислюються: When determining the energy expenditure of the human body, it was established that the respiratory coefficient is equal to 1.0. This means that the cells of the subject are mainly oxidized:

Вуглеводи Carbohydrates

Жири Fats

Білки та вуглеводи Proteins and carbohydrates

Білки Proteins

Вуглеводи та жири Carbohydrates and Fats

36 / 200
У добовому раціоні дорослої здорової людини повинні бути жири, білки, вуглеводи, вітаміни, мінеральні солі та вода. Вкажіть добову кількість білку, яка забезпечує нормальну життєдіяльність організму: The daily diet of a healthy adult should include fats, proteins, carbohydrates, vitamins, mineral salts and water. Specify the daily amount of protein that ensures the normal functioning of the body:

10-20 10-20

50-60 50-60

100-120 100-120

40-50 40-50

70-80 70-80

37 / 200
При обстеженні хворого виявлені дерматит, діарея, деменція. Відсутність якого вітаміну є причиною цього стану? During the examination of the patient, dermatitis, diarrhea, dementia were found. The lack of which vitamin is the cause of this condition?

Фолієва кислота Folic acid

Біотин Biotin

Аскорбінова кислота Ascorbic acid

Нікотинамід Nicotinamide

Рутин Routine

38 / 200
Аміак є дуже отруйною речовиною, особливо для нервової системи. Яка речовина бере особливо активну участь у знешкодженні аміаку в тканинах мозку? Ammonia is a very poisonous substance, especially for the nervous system. What substance is particularly active in neutralizing ammonia in brain tissue?

Глутамінова кислота Glutamic acid

Аспарагінова кислота Aspartic acid

Буферні системи Buffer systems

Аргінін Arginine

Сечовина Urea

39 / 200
При обстеженні хворого виявлено підвищено вмісту в сироватці крові ліпопротеїнів низької щільності. Яке захворювання можна передбачити у цього хворого? During the examination of the patient, an increased content of low-density lipoproteins in the blood serum was found. What disease can be predicted in this patient?

Запалення легень Pneumonia

Гострий панкреатит Acute pancreatitis

Ураження нирок Kidney damage

Атеросклероз Atherosclerosis

Гастрит Gastritis

40 / 200
У відділення інтенсивної терапії доставлено жінку 50-ти років з діагнозом: інфаркт міокарда. Активність якого ферменту буде найбільш підвищена впродовж перших двох діб? A 50-year-old woman was brought to the intensive care unit with a diagnosis of myocardial infarction. The activity of which enzyme will be increased the most during the first two days?

Лужна фосфатаза Alkaline phosphatase

Аспартатамінотрансфераза Aspartate aminotransferase

Аланінамінопептидаза Alanine aminopeptidase

Сорбітдегідрогеназа Sorbit dehydrogenase

Аланінамінотрансфераза Alanine aminotransferase

41 / 200
У лікарню поступила робітниця хімічного підприємства з ознаками отруєння. У волоссі цієї жінки знайдено підвищену концентрацію арсенату, який блокує ліпоєву кислоту. Порушення якого процесу є найімовірною причиною порушень в організмі? A worker at a chemical plant was admitted to the hospital with signs of poisoning. An increased concentration of arsenate, which blocks lipoic acid, was found in this woman's hair. Disturbance of which process is the most likely cause of disorders in the body ?

Відновлення метгемоглобіну Recovery of methemoglobin

Знешкодження супероксидних іонів Detoxification of superoxide ions

Відновлення органічних перекисей Recovery of organic peroxides

Окислювальне декарбоксилювання ПВК Oxidative decarboxylation of PVC

Мікросомальне окислення Microsomal oxidation

42 / 200
При захворюваннях підшлункової залози порушується утворення та секреція трипсину. Назвіть речовини, травлення яких буде порушене? In diseases of the pancreas, the formation and secretion of trypsin is disturbed. Name the substances whose digestion will be disturbed?

Вуглеводи Carbohydrates

Нуклеїнові кислоти Nucleic acids

Фосфоліпіди Phospholipids

Білки Proteins

Ліпіди Lipids

43 / 200
Еритроцити людини не містять мітохондрій. Який основний шлях утворення АТФ в цих клітинах? Human erythrocytes do not contain mitochondria. What is the main way of generating ATP in these cells?

Окиснювальне фосфорилювання Oxidative Phosphorylation

Анаеробний гліколіз Anaerobic glycolysis

Креатинкіназна реакція Creatine kinase reaction

Аеробний гліколіз Aerobic glycolysis

Аденілаткіназна реакція Adenylate kinase reaction

44 / 200
Катіонні глікопротеїни є основними компонентами слини привушних залоз. Які амінокислоти обумовлюють їх позитивний заряд? Cationic glycoproteins are the main components of parotid gland saliva. What amino acids determine their positive charge?

Лізин, аргінін, гістидин Lysine, arginine, histidine

Аспартат, аргінін, глутамат Aspartate, arginine, glutamate

Глутамат, валін, лейцин Glutamate, valine, leucine

Цистеїн, гліцин, пролін Cysteine, glycine, proline

Аспартат, глутамат, гліцин Aspartate, glutamate, glycine

45 / 200
Однією із функцій слини є захисна, яка реалізується, зокрема, формуванням місцевого імунітету слизової оболонки за рахунок виділення привушними залозами такого білку: One of the functions of saliva is protective, which is implemented, in particular, by the formation of local immunity of the mucous membrane due to the release of such a protein by the parotid glands:

Секреторний імуноглобулін А Secretory immunoglobulin A

Еластин Elastin

Альбумін Albumin

Колаген Collagen

Фібриноген Fibrinogen

46 / 200
У хлопчика 4-х років після перенесеного важкого вірусного гепатиту спостерігаються блювання, епізоди непритомності, судоми. У крові - гіперамоніємія. Порушення якого біохімічного процесу в печінці викликало такий стан хворого? A 4-year-old boy has vomiting, episodes of fainting, and convulsions after suffering severe viral hepatitis. Hyperammonemia is present in the blood. What biochemical process in the liver was disturbed that caused this condition the patient?

Активація декарбоксилювання амінокислот Activation of decarboxylation of amino acids

Пригнічення ферментів трансамінуван-ня Inhibition of transamination enzymes

Пригнічення синтезу білків Inhibition of protein synthesis

Порушення знешкодження біогенних амінів Violation of neutralization of biogenic amines

Порушення знешкодження аміаку Ammonia neutralization violation

47 / 200
Хворому з печінковою недостатністю проведено дослідження електрофоретичного спектру білків сироватки крові. Які фізико-хімічні властивості білкових молекул лежать в основі цього методу? The electrophoretic spectrum of blood serum proteins was studied in a patient with liver failure. What physical and chemical properties of protein molecules are the basis of this method?

Оптична активність Optical activity

Гідрофільність Hydrophilicity

Здатність набрякати Ability to swell

Наявність заряду Availability of charge

Нездатність до діалізу Inability to dialysis

48 / 200
При спадковій оратацидурії виділення оротової кислоти в багато разів перевищує норму. Синтез яких речовин буде порушений при цій патології? With hereditary orataciduria, the excretion of orotic acid is many times higher than the norm. The synthesis of what substances will be disturbed in this pathology?

Піримідинові нуклеотиди Pyrimidine nucleotides

Сечовина Urea

Сечова кислота Uric acid

Біогенні аміни Biogenic amines

Пуринові нуклеотиди Purine nucleotides

49 / 200
У хворого 37-ми років після автомобільної катастрофи АТ- 70/40 мм рт.ст. Хворий у непритомному стані. За добу виділяє близько 300 мл сечі. Який механізм порушення сечоутворення у даному випадку? A 37-year-old patient after a car accident has blood pressure of 70/40 mmHg. The patient is unconscious. He excretes about 300 ml of urine per day. What the mechanism of urinary disorders in this case?

Зменшення канальцевої секреції Reduction of tubular secretion

Зменшення канальцевої реабсорбції Reduction of tubular reabsorption

Посилення клубочкової фільтрації Increase glomerular filtration

Посилення канальцевої реабсорбції Strengthening of tubular reabsorption

Зменшення клубочкової фільтрації Decreasing glomerular filtration

50 / 200
У хворого, якому було екстирповано зуб з приводу гострого гнійного періоститу, спостерігалася тривала кровотеча з комірки, яку не можна було спинити звичайними методами. У крові: ер.-2, 9 • 1012/л, НЬ- 90 г/л; КП- 0,9; тромб.-60 • 109/л; лейк.- 52 • 109/л.; базофіли - 0, еозинофіли -1%, моноцити - 0, нейтрофіли: юні - 0, паличкоядерні - 2%, сегменто-ядерні -18%, лімфоцити - 8%, моноцити -1%, мієлобласти - 70%. Яке захворювання крові наявне у цього хворого? A patient who had a tooth extracted due to acute purulent periostitis had prolonged bleeding from a cell that could not be stopped by conventional methods. In the blood: er.- 2, 9 • 1012/l, Hb- 90 g/l, CP- 0.9, thrombocyte-60 • 109/l, leuk.- 52 • 109/l, basophils - 0, eosinophils -1%, monocytes - 0, neutrophils: young - 0, rod-nuclear - 2%, segmento-nuclear - 18%, lymphocytes - 8%, monocytes - 1%, myeloblasts - 70%. What blood disease does this patient have?

Недиференційований лейкоз Undifferentiated leukemia

Хронічний мієлоїдний лейкоз Chronic myeloid leukemia

Гострий мієлоїдний лейкоз Acute myeloid leukemia

Промієлоцитарний лейкоз Promyelocytic leukemia

Еритромієлоз Erythromyelosis

51 / 200
Жінка із токсикозом вагітності страждає на гіперсалівацію, що призводить до втрати 3-4 літрів слини щоденно. Яке порушення водно-сольового обміну виникає при цьому? A woman with toxicosis of pregnancy suffers from hypersalivation, which leads to the loss of 3-4 liters of saliva daily. What kind of violation of water-salt metabolism occurs in this case?

Гіпогідратація гіперосмолярна Hyporosmolar hypohydration

Гіпонатріємія Hyponatremia

Гіпокаліємія Hypokalemia

Гіпогідратація ізоосмолярна Hypohydration isosmolar

Гіпогідратація гіпоосмолярна Hypohydration hypoosmolar

52 / 200
У водолаза, що проводив роботи на великій глибині, при швидкому поверненні в умови нормального атмосферного тиску з’явилися біль у суглобах, свербіж шкіри, порушення зору, непритомність. Як називається описане явище? The diver, who was working at a great depth, experienced pain in the joints, itchy skin, impaired vision, and fainting when he quickly returned to normal atmospheric pressure. What is the name of the described phenomenon?

Гіпероксія Hyperoxia

Хвороба декомпресії Decompression sickness

Синдром вибухової декомпресії Explosive decompression syndrome

Баротравма Barotrauma

Стан невагомості State of weightlessness

53 / 200
Через 8 днів після опромінення у ліквідатора аварії на АЕС розвинулись виразково- некротичні зміни у порожнині рота. У крові: ер- 3,2 • 1012/л, ретикулоцитів 0,01%, НЬ- 60 г/л, лейк.- 2,3 • 109/л, тромбоцитів 50 тис/л. Для якого періоду променевої хвороби характерні описані зміни? 8 days after exposure, the liquidator of the accident at the nuclear power plant developed ulcerative-necrotic changes in the oral cavity. In the blood: er- 3.2 • 1012/l, reticulocytes 0.01%, Hb- 60 g/l, leuk.- 2.3 • 109/l, platelets 50 thousand/l. For what period of radiation sickness are the described changes typical?

Закінчення хвороби End of illness

Прихований період Hidden period

Період уявного благополуччя Period of imaginary well-being

Період первинних реакцій Period of primary reactions

Період розвинутих клінічних ознак Period of developed clinical signs

54 / 200
Епідеміологічне дослідження розповсюдження пухлин виявило високу кореляцію розвитку пухлин легень з тютюнопалінням. З дією якого хімічного канцерогену найбільш імовірно пов’язано виникнення даного виду патології? An epidemiological study of the spread of tumors revealed a high correlation between the development of lung tumors and tobacco smoking. Which chemical carcinogen is most likely associated with the occurrence of this type of pathology?

Ортоаміноазотолуол Orthoaminoazotoluene

Метилхолантрен Methylcholanthrene

3,4-бензпірен 3,4-benzpyrene

Афлатоксин Aflatoxin

Діетілнітрозамін Diethylnitrosamine

55 / 200
У хворого виявлено ожиріння, гірсутизм, 'місяцеподібне' обличчя, рубці багряного кольору на шкірі стегон. АТ-180/110 мм рт.ст., глюкоза крові - 17,2 ммоль/л. При якій зміні секреції гормонів наднирників можлива така картина? The patient was found to have obesity, hirsutism, a 'moon-shaped' face, purple scars on the skin of the thighs. BP-180/110 mm Hg, blood glucose - 17.2 mmol/l. At what change in the secretion of adrenal hormones is such a picture possible?

Гіперсекреція глюкокортикоїдів Hypersecretion of glucocorticoids

Гіперсекреція мінералокортикоїдів Hypersecretion of mineralocorticoids

Гіпосекреція мінералокортикоїдів Hyposecretion of mineralocorticoids

Гіпосекреція адреналіну Hyposecretion of adrenaline

Гіпосекреція глюкокортикоїдів Glucocorticoid hyposecretion

56 / 200
При мікроскопічному дослідження пунктату з осередку запалення у хворого із абсцесом шкіри знайдено велику кількість різних клітин крові. Які з цих клітин першими надходять із судин до тканин при запаленні? During a microscopic examination of a punctate from a focus of inflammation in a patient with a skin abscess, a large number of different blood cells were found. Which of these cells are the first to arrive from vessels to tissues during inflammation?

Базофіли Basophils

Моноцити Monocytes

Еозинофіли Eosinophils

Лімфоцити Lymphocytes

Нейтрофіли Neutrophils

57 / 200
У жінки, яка протягом 15-ти років страждала на виражену гіпертензію, останнім часом з’явились задишка, серцебиття, дещо знизився систолічний тиск. Який основний механізм виникнення у хворої серцевої недостатності? A woman who has suffered from severe hypertension for 15 years has recently developed shortness of breath, palpitations, and her systolic pressure has decreased somewhat. What is the main mechanism of occurrence in a patient with heart failure?

Ушкодження міокарда Myocardial damage

Порушення регуляції серцевої діяльності Cardiac dysregulation

Перевантаження серця збільшеним об’ємом крові Overload of the heart with increased blood volume

Порушення проведення імпульсу по міокарду Violation of impulse conduction through the myocardium

Перевантаження серця збільшеним опором викиду крові Overload of the heart with increased resistance to blood ejection

58 / 200
При гістологічному дослідженні стінки кісти, що локалізується у ділянці верхньої щелепи, встановлено, що стінка кісти з середини вистелена багатошаровим плоским епітелієм з підлеглою грануляційною тканиною з лімфолейкоцитарною інфільтрацією. Зовнішній шар представлений пухкою волокнистою сполучною тканиною, оточеною рубцевою фіброзною тканиною. Ці дані є підставою для встановлення такого діагнозу: During the histological examination of the wall of the cyst localized in the area of the upper jaw, it was established that the wall of the cyst is lined from the middle with a multilayered flat epithelium with underlying granulation tissue with lymphocytic infiltration. External the layer is represented by loose fibrous connective tissue surrounded by scar tissue. These data are the basis for establishing the following diagnosis:

Проста гранульома Simple granuloma

Амелобластома Ameloblastoma

Кератокіста Keratocyst

Кістогранульома Cystogranuloma

Епітеліальна гранульома Epithelial granuloma

59 / 200
При обстеженні у хворого на твердому піднебінні виявлена пухлина у вигляді невеликого щільного вузла сірого кольору без чітких меж. Після видалення пухлина досліджена гістологічно. Побудована з дрібних, кубічної форми клітин з гіперхромним ядром, які формують альвеоли, трабекули, солідні та кріброзні структури. Ріст пухлини - інвазивний. Назвіть пухлину: During the examination, a tumor was found on the patient's hard palate in the form of a small dense gray nodule without clear boundaries. After removal, the tumor was examined histologically. It is made of small, cubic-shaped cells with a hyperchromic nucleus, which form alveoli, trabeculae, solid and cribrous structures. Tumor growth is invasive. Name the tumor:

Аденолімфома Adenolymphoma

Мукоепідермоїдний рак Mucoepidermoid cancer

Аденокістозна карцинома Adenocystic carcinoma

Мономорфна аденома Monomorphic adenoma

Злоякісна плеоморфна аденома Malignant pleomorphic adenoma

60 / 200
В біоптаті слизової носа знайдені епітеліоїдні клітини, плазмоцити, клітини Мікулича, еозинофільні тільця Русселя. Який найбільш імовірний діагноз? Epithelioid cells, plasma cells, Mikulich cells, eosinophilic Russel bodies were found in the biopsy of the nasal mucosa. What is the most likely diagnosis?

Алергічний риніт Allergic rhinitis

Респіраторно-синцитіальна інфекція Respiratory syncytial infection

Туберкульоз Tuberculosis

Сифіліс Syphilis

Риносклерома Rhinoscleroma

61 / 200
Хворий 58-ми років впродовж багатьох років страждав на хронічний остеомієліт нижньої щелепи. Останнім часом в аналізі крові - гіпопротеїнемія, диспротеїнемія; у сечі - протеїнурія, білкові циліндри. Помер від хронічної ниркової недостатності. На розтині нирки збільшені, щільні, воскоподібні. Який патологічний процес у нирках був виявлений на розтині? A 58-year-old patient has been suffering from chronic osteomyelitis of the lower jaw for many years. Recently, in the blood test - hypoproteinemia, dysproteinemia; in the urine - proteinuria, protein cylinders. He died of chronic kidney failure. At autopsy, the kidneys are enlarged, dense, wax-like. What pathological process in the kidneys was revealed at autopsy?

Гідронефроз Hydronephrosis

Хронічний гломерулонефрит Chronic glomerulonephritis

Амілоїдоз Amyloidosis

Хронічний пієлонефрит Chronic pyelonephritis

Інтерстиційний нефрит Interstitial jade

62 / 200
У хворого 77-ми років, що страждає на атеросклероз, з’явився біль у правій ступні. Ступня збільшена в розмірі, шкірні покриви чорного кольору, мацеровані, демаркаційна зона невиражена. Який патологічний процес у ступні діагностовано? A 77-year-old patient suffering from atherosclerosis developed pain in the right foot. The foot is enlarged, the skin is black, macerated, demarcation the zone is not expressed. What pathological process is diagnosed in the foot?

Нома Noma

Коагуляційний некроз Coagulation necrosis

Суха гангрена Dry gangrene

Волога гангрена Wet gangrene

Секвестр Sequestration

63 / 200
При огляді хворого лікар-стоматолог відмітив, що краї зубної комірки, позбавленої зуба, значно зближені, діаметр її зменшений. Про який загальнопатологічний процес йдеться в даному випадку? During the examination of the patient, the dentist noticed that the edges of the tooth cell, deprived of a tooth, are significantly closer together, its diameter is reduced. What general pathological process is this case about?

Атрофія, викликана недостатністю кровообігу Atrophy caused by lack of blood circulation

Гіпоплазія Hypoplasia

Атрофія нейротична (при денервації) Neurotic atrophy (with denervation)

Атрофія від тиску Atrophy from pressure

Атрофія дисфункціональна Atrophy is dysfunctional

64 / 200
Чоловік віком 42 роки помер при явищах вираженої інтоксикації та дихальної недостатності. На розрізі тканина легень у всіх відділах строката, з множинними дрібновогнищевими крововиливами та вогнищами емфіземи. Гістологічно у легенях: геморагічна бронхопневмонія з абсцедуванням, у цитоплазмі клітин епітелію бронхів еозинофільні та базофільні включення. Який найбільш імовірний діагноз? A 42-year-old man died due to symptoms of severe intoxication and respiratory failure. On cross-section, the lung tissue in all sections is variegated, with multiple small focal hemorrhages and foci of emphysema. Histologically, the lungs : hemorrhagic bronchopneumonia with abscessation, eosinophilic and basophilic inclusions in the cytoplasm of bronchial epithelial cells. What is the most likely diagnosis?

Стафілококова бронхопневмонія Staphylococcal bronchopneumonia

Грип Flu

Парагрип Paraflu

Респіраторно-синцитіальна інфекція Respiratory syncytial infection

Аденовірусна інфекція Adenovirus infection

65 / 200
Макроскопічно печінка збільшена в розмірах, ущільнена, тканина сірувато-жовтого кольору, з сальним блиском. Який патологічний процес лежить в основі описаних змін? Macroscopically, the liver is enlarged, compacted, the tissue is grayish-yellow, with a greasy sheen. What pathological process is the basis of the described changes?

Гіаліноз Hyalinosis

Мукоїдне набухання Mucoid swelling

Жирова дистрофія Fat dystrophy

Гемахроматоз Hemachromatosis

Амілоїдоз Amyloidosis

66 / 200
При дослідженні біоптату збільшеного шийного лімфатичного вузла виявлена стертість малюнка, тканина його представлена великою кількістю проліферуючих лімфоцитів з домішкою поодиноких клітин Березовського-Штернберга. Ці зміни свідчать про таке захворювання: During examination of the biopsy of an enlarged cervical lymph node, a roughness of the pattern was found, its tissue is represented by a large number of proliferating lymphocytes with an admixture of solitary Berezovsky-Sternberg cells. These changes indicate the following disease:

Лімфосаркома Lymphosarcoma

Лімфогранулематоз з переважанням лімфоїдної тканини Lymphogranulomatosis with predominance of lymphoid tissue

Змішано-клітинний варіант лімфогранулематозу Mixed cell variant of lymphogranulomatosis

Нодулярно-склеротичний варіант лімфогранулематозу Nodular-sclerotic variant of lymphogranulomatosis

Лімфогранулематоз з виснаженням лімфоїдної тканини Lymphogranulomatosis with depletion of lymphoid tissue

67 / 200
У жінки 45-ти років, яка померла від хронічної алкогольної інтоксикації, на аутопсії печінка різко збільшена, тістоподібної консистенції, жовтуватого кольору. Мікроскопічно в цитоплазмі гепатоцитів при фарбуванні гематоксиліном та еозином виявляються різних розмірів оптично порожні вакуолі. Який вид дистрофії має місце? In a 45-year-old woman who died of chronic alcohol intoxication, at autopsy, the liver was sharply enlarged, had a pasty consistency, and was yellowish in color. Microscopically, in the cytoplasm of hepatocytes when stained with hematoxylin and eosin reveal optically empty vacuoles of various sizes. What kind of dystrophy is taking place?

Паренхіматозна жирова Parenchymatous fatty

Мезенхімальна жирова Mesenchymal adipose

Гіаліново-крапельна Hyaline-drip

Вуглеводна паренхіматозна Carbohydrate parenchymatous

Гідропічна Hydropic

68 / 200
У хворого 23-х років після перенесеної ангіни розвинувся сечовий синдром (гематурія, протеїнурія, лейкоцитурія). В пункційному біоптаті нирок виявлена картина інтракапілярного проліферативного гломерулонефриту, а електронно-мікроскопічно виявлені великі субепітеліальні депозити. Який патогенез цього захворювання? A 23-year-old patient developed a urinary syndrome (hematuria, proteinuria, leukocyturia) after a sore throat. A puncture biopsy of the kidneys revealed intracapillary proliferative glomerulonephritis, and electronic large subepithelial deposits were detected microscopically. What is the pathogenesis of this disease?

Клітинно обумовлений цитоліз Cell-induced cytolysis

Цитотоксична, цитолітична дія антитіл Cytotoxic, cytolytic effect of antibodies

Атопія, анафілаксія з утворенням IgE і фіксацією їх на опасистих клітинах Atopy, anaphylaxis with the formation of IgE and their fixation on mast cells

Гранулематоз Granulomatosis

Імунокомплексний механізм Immune complex mechanism

69 / 200
З випорожнень хворої дитини 6-місячного віку, яка знаходиться на штучному вигодовуванні, виділена культура кишкової палички з антигенною структурою 0-111. Який діагноз можна поставити? Escherichia coli culture with antigenic structure 0-111 was isolated from the feces of a sick 6-month-old child who is on artificial feeding. What diagnosis can be made?

Харчове отруєння Food poisoning

Колі-ентерит Coli-enteritis

Холероподібне захворювання Cholera-like disease

Дизентерієподібне захворювання Dysentery-like disease

Гастроентерит Gastroenteritis

70 / 200
До інфекційного відділення госпіталізовано хворого 27-ми років зі скаргами на багаторазові пронос та блювання, біль у м’язах ніг, слабкість, запаморочення. Після огляду лікар поставив попередній діагноз 'холера'. Як необхідно досліджувати матеріал від хворого для експрес діагнозу? A 27-year-old patient was hospitalized in the infectious department with complaints of repeated diarrhea and vomiting, pain in the muscles of the legs, weakness, dizziness. After the examination, the doctor put the previous 'Cholera diagnosis. How should the patient's material be examined for an express diagnosis?

Серологічний метод Serological method

Пряма і непряма РІФ Direct and indirect RIF

Біологічний метод Biological method

РА RA

Бактеріологічний метод Bacteriological method

71 / 200
З метою підтвердження діагнозу туляремії мисливцю, госпіталізованому на 5-й день хвороби, слід призначити для ранньої діагностики: In order to confirm the diagnosis of tularemia, a hunter hospitalized on the 5th day of illness should be prescribed for early diagnosis:

Алергічна проба Allergy test

Реакція іммунофлюоресценції Immunofluorescence reaction

Реакція непрямої гемаглютинації Indirect hemagglutination reaction

Реакція аглютинації Agglutination reaction

Реакція зв’язування комплементу Complement binding reaction

72 / 200
Хворому на туберкульоз 34-х років, в анамнезі якого була відкрита легенева форма захворювання, проведено мікроскопічне дослідження харкотиння з метою виявлення збудника. Який метод забарвлення доцільно використати при цьому? A 34-year-old tuberculosis patient with a history of an open pulmonary form of the disease underwent a microscopic examination of sputum to identify the causative agent. What staining method should be used in this case ?

Метод Ціля-Нільсена Tiel-Nielsen Method

Метод Романовського-Гімзи Romanovsky-Giemsa method

Метод Буррі-Гінса Burry-Guins Method

Метод Нейссера Neisser Method

Метод Грама Gram's method

73 / 200
Після посіву гною з уретри на спеціальне поживне середовище виросли ніжні блакитнуваті колонії. При мікроскопії препаратів з них виявленні грамнегативні бобоподібні диплококи. Збудником якої хвороби вони є? After inoculation of pus from the urethra on a special nutrient medium, delicate bluish colonies grew. Gram-negative leguminous diplococci were detected during microscopy of preparations from them. What disease are they the causative agent of?

Сифіліс Syphilis

Туляремія Tularemia

Меліоїдоз Melioidosis

Хламідіоз Chlamydia

Гонорея Gonorrhea

74 / 200
У дитячому відділенні інфекційної клініки хлопчику 4-х років поставлено діагноз 'дифтерія'. Який препарат потрібно ввести хворому в першу чергу? In the children's department of the infectious disease clinic, a 4-year-old boy was diagnosed with diphtheria. What drug should be administered to the patient first?

АКДП AKDP

АДП ADP

Протидифтерійна антитоксична сироватка Anti-diphtheria anti-toxic serum

Дифтерійний анатоксин Diphtheria toxoid

ТАВї Tavi

75 / 200
У пацієнта з попереднім діагнозом 'сифіліс' лаборант взяв сироватку крові для постановки імунної реакції, яка основана на виявленні антитіл, які припиняють рух трепонем і призводять до їх загибелі. Яку реакцію було використано для діагностики? From a patient with a previous diagnosis of syphilis, a laboratory technician took blood serum to test an immune reaction, which is based on the detection of antibodies that stop the movement of treponema and lead to their death. What reaction was used for diagnosis?

Реакція аглютинації Agglutination reaction

Реакція нейтралізації Neutralization reaction

Реакція преципітації Precipitation reaction

Реакція іммобілізації Immobilization reaction

Реакція зв’язування комплементу Complement binding reaction

76 / 200
При повторній постановці реакції аглютинації Відаля виявлено наростання у сироватці хворого титрів антитіл до О-антигенів S. typhi з 1:100 до 1:400. Як можна розцінити отримані результати? When repeating the Vidal agglutination reaction, an increase in titers of antibodies to O-antigens of S. typhi from 1:100 to 1:400 was detected in the patient's serum. How can the obtained results?

Раніше був щеплений проти черевного тифу Previously vaccinated against typhoid

Є хронічним носієм черевнотифозних мікробів Is a chronic carrier of typhoid germs

Є гострим носієм черевнотифозних мікробів Is an acute carrier of typhoid germs

Хворіє на черевний тиф Sick of typhoid fever

Раніше перехворів на черевний тиф Previously fell ill with typhoid

77 / 200
При обстеженні на бактеріоносійство черевного тифу у сироватці крові кухарки шкільної їдальні виявлені Vi -антитіла. Яка з наведених реакцій була використана у даному випадку? Vi-antibodies were detected in the blood serum of the school canteen cook during the examination for typhoid bacteria. Which of the following reactions was used in this case?

Імуноферментний аналіз Enzyme immunoassay

Реакція Відаля Vidal reaction

Реакція зв’язування комплементу Complement binding reaction

Реакція імунофлюоресценції Immunofluorescence reaction

Реакція непрямої гемаглютинації Indirect hemagglutination reaction

78 / 200
Хворий з гіпертонічною хворобою II ст. за рекомендацією лікаря почав приймати антигіпертензивний препарат. Через кілька днів прийому препарату артеріальній тиск нормалізувався, але хворий помітив, що у нього з’явився сухий кашель. Який препарат був призначений? A patient with hypertension of the II stage started taking an antihypertensive drug on the recommendation of a doctor. After a few days of taking the drug, blood pressure normalized, but the patient noticed that he had a dry cough appeared. What drug was prescribed?

Клофелін Clofelin

Верапаміл Verapamil

Пропранолол (анаприлін) Propranolol (anaprilin)

Резерпін Reserpin

Еналапріл Enalapril

79 / 200
У хворої з артеріальною гіпертензією розвинувся напад бронхіальної астми. Який з нижче перерахованих бронхолітичних засобів може спровокувати гіпертонічний криз? A patient with arterial hypertension developed an attack of bronchial asthma. Which of the following bronchodilator agents can provoke a hypertensive crisis?

Ізадрин Izadrin

Ефедрину гідрохлорид Ephedrine hydrochloride

Еуфілін Euphilin

Сальбутамол Salbutamol

Беротек Berotek

80 / 200
Для корекції артеріального тиску при колаптоїдному стані хворому було введено мезатон. Який механізм дії даного препарату? Mezaton was administered to the patient to correct blood pressure in a colaptoid condition. What is the mechanism of action of this drug?

Стимулює α- β-адренорецептори Stimulates α-β-adrenoceptors

Стимулює α-адренорецептори Stimulates α-adrenoceptors

Блокує β-адренорецептори Blocks β-adrenoceptors

Блокує α-адренорецептори Blocks α-adrenoceptors

Стимулює β-адренорецептори Stimulates β-adrenoceptors

81 / 200
Хворому 60-ти років було встановлено діагноз цироз печінки, причиною якого було токсичне ураження алкоголем. Який препарат краще за все призначити хворому? A 60-year-old patient was diagnosed with cirrhosis of the liver, the cause of which was toxic alcohol damage. What drug is best to prescribe to the patient?

Аскорбінова кислота Ascorbic acid

Кислота глютамінова Glutamic acid

Вітамін Е Vitamin E

Ессенціале Essential

Рібоксин Riboxin

82 / 200
Хворий звернувся до лікаря зі скаргами на стоматит, який був лікований антисептичними засобами. Який препарат необхідно призначити хворому, якщо ураження слизової оболонки мають герпетичну природу? The patient turned to the doctor with complaints of stomatitis, which was treated with antiseptic agents. What drug should be prescribed to the patient if the lesions of the mucous membrane are herpetic in nature?

Бісептол Biseptol

Ацикловір Acyclovir

Ремантадин Remantadine

Фуразолідон Furazolidone

Клотримазол Clotrimazole

83 / 200
Хворому 28-ми років на бактеріальну пневмонію призначили курс лікування еритроміцином. Відомо, що його антибактеріальні властивості зумовлені здатністю цієї речовини сполучатися з вільною 508-субодиницею рибосоми. Синтез яких речовин блокує цей антибіотик у бактеріальних клітинах? A 28-year-old patient was prescribed a course of treatment with erythromycin for bacterial pneumonia. It is known that its antibacterial properties are due to the ability of this substance to combine with the free 508-subunit of the ribosome. The synthesis of which What substances does this antibiotic block in bacterial cells?

Білки Proteins

Жири Fats

ДНК DNA

Полісахариди Polysaccharides

РНК RNA

84 / 200
Небезпечно витискувати гноячки на обличчі, особливо вище ротової щілини. Через анастомози з якими венами обличчя може поширитися інфекція у порожнину черепа? It is dangerous to squeeze pustules on the face, especially above the oral cavity. Through anastomoses with which veins of the face can the infection spread to the skull cavity?

Передні вушні вени Anterior ear veins

Потилична вена Occipital vein

Середня і глибока скроневі вени Medium and deep temporal veins

Очні вени Ocular veins

Вени крилоподібного сплетення Veins of the pterygoid plexus

85 / 200
Дівчинка 11-ти років часто хворіє на гострі респіраторні інфекції, після яких спостерігаються множинні точкові крововиливі в місцях тертя одягу. Гіповітаміноз якого вітаміну має місце в дівчинки? An 11-year-old girl often suffers from acute respiratory infections, after which multiple point hemorrhages are observed in places of friction of clothes. Hypovitaminosis of which vitamin occurs in the girl?

В2 B2

А A

Вб Vb

С С

В1 B1

86 / 200
Хворого доставили до клініки у коматозному стані. В анамнезі цукровий діабет 2 типу впродовж 5-ти років. Об’єктивно: дихання шумне, глибоке, у видихуваному повітрі відчувається запах ацетону. Вміст глюкози у крові 15,2 ммоль/л, кетонових тіл - 100 мкмоль/л. Для якого ускладнення даного захворювання характерні такі розлади? The patient was brought to the clinic in a comatose state. He has a history of type 2 diabetes for 5 years. Objectively: breathing is noisy, deep, in the exhaled air you can feel the smell of acetone. The content of glucose in the blood is 15.2 mmol/l, ketone bodies - 100 μmol/l. For which complication of this disease are such disorders characteristic?

Гіпоглікемічна кома Hypoglycemic coma

Гіперглікемічна кома Hyperglycemic coma

Гіперосмолярна кома Hyperosmolar coma

Печінкова кома Hepatic coma

Кетоацидотична кома Ketoacidotic coma

87 / 200
Хвора на хронічний гепатит скаржиться на підвищення чутливості до барбітуратів, які раніше вона переносила без симптомів інтоксикації. З порушенням якої функції печінки це пов’язане найбільше? A patient with chronic hepatitis complains of increased sensitivity to barbiturates, which she previously tolerated without symptoms of intoxication. Which liver function disorder is this most associated with?

Гемопоетична Hematopoietic

Фагоцитарна Phagocytic

Утворення жовчі Bile Formation

Гемодинамічна Hemodynamic

Метаболічна Metabolic

88 / 200
У лікарню потрапив хворий з високою температурою, маренням, розчухами на голові. На голові виявлені комахи, сірого кольору, довжиною 3 мм, із сплощеним у дорзовентральному напрямі тілом і трьома парами кінцівок. Для якої патології характерна описана картина? A patient was admitted to the hospital with a high temperature, delirium, scratches on the head. On the head, insects were found, gray in color, 3 mm long, with a body flattened in the dorsoventral direction and by three pairs of limbs. What pathology is the described picture characteristic of?

Скабієс Scabies

Педикульоз Pediculosis

Ураження шкіри клопами Skin bug infestation

Демодекоз Demodecosis

Алергія Allergy

89 / 200
Під час прийому їжі у новонародженої дитини спостерігається закидання молока у носову порожнину. Вкажіть на можливу причину виникнення цього порушення. While eating, a newborn baby is observed throwing milk into the nasal cavity. Indicate the possible cause of this disorder.

Викривлення носової перегородки вліво Curvation of the nasal septum to the left

Заяча губа Hare's lip

Вовча паща Wolf's Mouth

Викривлення носової перегородки вправо Curvation of the nasal septum to the right

Перелом основи черепа Skull base fracture

90 / 200
Під час виконання фізичного навантаження людина менш чутлива до болю. Причиною цього є активація: When performing physical activity, a person is less sensitive to pain. The reason for this is activation:

Функції щитоподібної залози Thyroid functions

Ноціцептивної системи Nociceptive system

Функції наднирників Adrenal functions

Симпато-адреналової системи Sympatho-adrenal system

Антиноціцептивної системи Antinociceptive system

91 / 200
За даними ВООЗ щорічно на Землі на малярію хворіють приблизно 250 млн. чоловік. Ця хвороба зустрічається переважно у тропічних і субтропічних областях. Межі її розповсюдження співпадають з ареалами комарів роду: According to the WHO, approximately 250 million people on Earth are ill with malaria every year. This disease is found mainly in tropical and subtropical regions. The limits of its distribution coincide with the ranges of mosquitoes of the genus :

Мансоніа Mansonia

Кулекс Culex

Аедес Aedes

Кулізета Kuliseta

Анофелес Anopheles

92 / 200
У хворого з патологією серцево-судинної системи розвинулись набряки на нижніх кінцівках. Який механізм розвитку серцевого набряку? A patient with a pathology of the cardiovascular system developed edema on the lower extremities. What is the mechanism of the development of cardiac edema?

Підвищення гідростатичного тиску на артеріальному кінці капіляру Increased hydrostatic pressure at the arterial end of the capillary

Підвищення онкотичного тиску Increased oncotic pressure

Підвищення гідростатичного тиску на венозному кінці капіляру Increased hydrostatic pressure at the venous end of the capillary

Зниження осмотичного тиску Decrease in osmotic pressure

Порушення лімфовідтоку Disruption of lymphatic drainage

93 / 200
У хворого з’явилися жовтушність шкіри, склер та слизових оболонок. У плазмі крові підвищений рівень загального білірубіну, в калі - стеркобіліну, в сечі - уробіліну. Який вид жовтяниці у хворого? The patient developed yellowing of the skin, sclera, and mucous membranes. The level of total bilirubin in the blood plasma, stercobilin in the feces, and urobilin in the urine is elevated. What type jaundice in the patient?

Хвороба Жільбера Gilbert's disease

Паренхіматозна Parenchymatous

Холестатична Cholestatic

Обтураційна Obstructive

Гемолітична Hemolytic

94 / 200
У хворого лікар діагностував тромбоз селезінкової артерії (інфаркт селезінки). Звідки відходить уражена артерія? The doctor diagnosed the patient with thrombosis of the splenic artery (infarction of the spleen). Where does the affected artery come from?

Нижня брижова артерія Inferior mesenteric artery

Загальна печінкова артерія Common hepatic artery

Черевний стовбур Abdominal trunk

Ліва шлункова артерія Left gastric artery

Верхня брижова артерія Superior mesenteric artery

95 / 200
У хворого на хронічний пієлонефрит після неконтрольованого прийому сечогінних засобів виникли екстрасистоли та болі у ділянці серця. При дослідженні крові виявлено гіпокаліємію. Який з перерахованих препаратів слід призначити? A patient with chronic pyelonephritis developed extrasystoles and pain in the heart area after uncontrolled intake of diuretics. A blood test revealed hypokalemia. Which of the listed drugs should be prescribed?

Калію перманганат Potassium permanganate

Кальцію хлорид Calcium chloride

Гідрохлортіазид Hydrochlorothiazide

Ретинолу ацетат Retinol Acetate

Аспаркам Asparkam

96 / 200
Хворому 30-ти років для лікування пневмонії лікар на 3 дні призначив антибіотик з групи азалідів, що має бактерицидну дію, тривалий ефект, здатність зв’язуватись з фагоцитами і накопичуватись у вогнищах інфекції. Який препарат було призначено хворому? A 30-year-old patient was prescribed an antibiotic from the azalide group for 3 days to treat pneumonia, which has a bactericidal effect, a long-lasting effect, the ability to bind to phagocytes and accumulate in foci of infection. What drug was prescribed to the patient?

Бензилпеніциліну натрієва сіль Benzylpenicillin sodium salt

Ципрофлоксацин Ciprofloxacin

Азитроміцин Azithromycin

Ізоніазид Isoniazid

Еритроміцин Erythromycin

97 / 200
У хворого 64-х років часті напади стенокардії. Два роки тому він переніс інфаркт міокарда. Вкажіть групу лікарських засобів, препарати якої доцільно призначити для профілактики розвитку повторного інфаркту міокарда: A 64-year-old patient has frequent attacks of angina pectoris. Two years ago he suffered a myocardial infarction. Specify a group of drugs that should be prescribed to prevent the development of a repeated myocardial infarction :

Антиагреганти Antiplatelets

Антикоагулянти непрямої дії Anticoagulants of indirect action

Коагулянти Coagulants

Антагоністи гепарину Heparin antagonists

Фібринолітики Fibrinolytics

98 / 200
У печінці хворого, який страждає на залізодефіцитну анемію виявлено порушення синтезу залізовмісного білка, який є джерелом заліза для синтезу гему. Як називається цей білок? In the liver of a patient suffering from iron deficiency anemia, a violation of the synthesis of an iron-containing protein, which is a source of iron for the synthesis of heme, was detected. What is the name of this protein?

Феритин Ferritin

Церулоплазмін Ceruloplasmin

Трансферин Transferin

Гемосидерин Hemosiderin

Гемоглобін Hemoglobin

99 / 200
У ході клінічного обстеження пацієнта виявлено збільшення щитоподібної залози (зоб), підвищення основного обміну, втрата маси тіла, порушення теплового балансу, підвищення апетиту, підвищення збудливості та дратівливості, екзофтальм і тахікардія. Яке ендокринне порушення призводить до появи даних симптомів? In the course of the patient's clinical examination, an increase in the thyroid gland (goiter), an increase in the basal metabolism, a loss of body weight, a violation of the thermal balance, an increase in appetite, an increase in excitability and irritability, exophthalmos and tachycardia. What endocrine disorder leads to the appearance of these symptoms?

Гіпофункція щитоподібної залози Hypothyroidism

Гіперфункція щитоподібної залози Hyperthyroidism

Гіперфункція гіпофізу Hyperfunction of the pituitary gland

Гіпофункція паращитоподібних залоз Hypofunction of parathyroid glands

Гіпофункція епіфізу Pineal hypofunction

100 / 200
При тестуванні на гіперчутливість пацієнту під шкіру ввели алерген, після чого спостерігалось почервоніння, набряк, біль внаслідок дії гістаміну. В результаті якого перетворення амінокислоти гістидину утворюється цей біогенний амін? When testing for hypersensitivity, an allergen was injected under the patient's skin, after which redness, swelling, and pain were observed due to the action of histamine. As a result of which transformation of the amino acid histidine, this biogenic amine is formed?

Дезамінування Demining

Декарбоксилювання Decarboxylation

Ізомеризація Isomerization

Фосфорилювання Phosphorylation

Метилювання Methylation

101 / 200
У дитини 12-ти років низький зріст при непропорційній будові тіла і розумовій відсталості. Недостатня секреція якого гормону може бути причиною таких порушень? A 12-year-old child has short stature with a disproportionate body structure and mental retardation. Insufficient secretion of which hormone can be the cause of such disorders?

Кортизол Cortisol

Соматотропін Somatotropin

Тироксин Tyroxin

Глюкагон Glucagon

Інсулін Insulin

102 / 200
У реанімаційному відділенні знаходиться хворий 49-ти років у коматозному стані. При дослідженні крові відзначено збільшення концентрації К+, зменшення вмісту Сa++, ацидоз, збільшення рівня сечовини, сечової кислоти. Який вид коми за етіологією найбільш імовірний? In the intensive care unit there is a 49-year-old patient in a comatose state. Blood tests showed an increase in the concentration of K+, a decrease in the content of Ca++, acidosis, an increase in the level of urea, acids. What type of coma is most probable by etiology?

Діабетична Diabetic

Нейрогенна Neurogenic

Гіпоглікемічна Hypoglycemic

Ниркова Kidney

Печінкова Hepatic

103 / 200
До приймального відділення госпіталізовано чоловіка у непритомному стані. На зовнішні подразники не реагує, дихання періодичне по типу Чейн-Стокса, зіниці звужені, зіничний рефлекс відсутній. Було встановлено, що дані симптоми обумовлені застосуванням морфіну. Призначити антидотну терапію: A man was hospitalized in an unconscious state. He does not react to external stimuli, breathing is periodic according to the Cheyne-Stokes type, the pupils are narrowed, the pupillary reflex is absent. It was established that that these symptoms are due to the use of morphine. Prescribe antidote therapy:

Апоморфіну гідрохлорид Apomorphine hydrochloride

Кальцію хлорид Calcium chloride

Унітіол Unithiol

Протаміну сульфат Protamine sulfate

Налоксон Naloxone

104 / 200
У хворого із цирозом печінки відмічається стійка артеріальна гіпотензія. (АТ- 90/50 мм рт.ст.). Чим обумовлено зниження артеріального тиску при такій патології печінки? A patient with cirrhosis of the liver has sustained arterial hypotension. (BP - 90/50 mmHg). What is the reason for the decrease in arterial pressure in such liver pathology?

Активація калікреїн-кінінової системи Activation of the kallikrein-kinin system

Надмірна інактивація вазопресину Excessive vasopressin inactivation

Зниження синтезу ангіотензиногену Decreased synthesis of angiotensinogen

Посилення рефлекторного впливу із рецепторної зони дуги аорти Intensification of the reflex effect from the receptor zone of the aortic arch

Збільшення синтезу Na-уретичного гормону Increased Na-uretic hormone synthesis

105 / 200
Пацієнт доставлений до лікарні з симптомами запаморочення, сухості в роті, зіниці сильно розширені, порушення акомодації, тахікардія, утруднення сечовипускання, атонія кишечнику. Передозування яким препаратом могло викликати дані симптоми? The patient was brought to the hospital with symptoms of dizziness, dry mouth, greatly dilated pupils, impaired accommodation, tachycardia, difficulty urinating, intestinal atony. An overdose of which drug could have caused the data symptoms?

Клофелін Clofelin

Атропіну сульфат Atropine sulfate

Празозин Prazozin

Каптоприл Captopril

Фуросемід Furosemide

106 / 200
Який з легеневих об’ємів НЕМОЖЛИВО визначити за допомогою спірометрії? Which of the lung volumes CANNOT be determined using spirometry?

Дихальний об’єм Respiratory volume

Резервний об’єм вдиху Inspiratory reserve volume

Резервний об’єм видиху Reserve expiratory volume

Життєва ємність легенів Vital lung capacity

Залишковий об’єм Remaining Volume

107 / 200
При обстеженні хворого з ендокринною патологією встановлено, що в плазмі крові підвищений рівень тестостерону. Які клітини в організмі чоловіка відповідальні за продукцію цього гормону? During the examination of a patient with endocrine pathology, it was established that the level of testosterone in the blood plasma is elevated. Which cells in a man's body are responsible for the production of this hormone?

Клітини передміхурової залози Prostate cells

Гландулоцити сім’яників Glandulocytes of testes

Сустентоцити сім’яників Sustentocytes of testes

Сперматогенні клітини Spermatogenic cells

Клітини сім’яних міхурців Seminal vesicle cells

108 / 200
У хворого на енцефаліт спостерігаються дихальні рухи постійної амплітуди, які чергуються з періодами апное. Якому типу дихання це притаманно? A patient with encephalitis has respiratory movements of constant amplitude, which alternate with periods of apnea. What type of breathing is this characteristic of?

Куссмауля Kussmaul

Чейн-Стокса Chain-Stokes

Біота Biota

Апнейстичне Apneistic

Стенотичне Stenotic

109 / 200
Чоловік 60-ти років страждає на атеросклероз судин головного мозку. При обстеженні виявлена гіперліпідемія. Вміст якого класу ліпопротеїдів найбільш імовірно буде підвищений при дослідженні сироватки крові? A 60-year-old man suffers from atherosclerosis of cerebral vessels. During the examination, hyperlipidemia was detected. The content of which class of lipoproteins is most likely to be increased during the blood serum examination?

Комплекси жирних кислот з альбумінами Complexes of fatty acids with albumins

Ліпопротеїди низької щільності Low-density lipoprotein

Ліпопротеїди високої щільності High-density lipoprotein

Холестерин Cholesterol

Хіломікрони Chylomicrons

110 / 200
Для покращання трофіки серцевого м’яза пацієнту призначено кардонат, до складу якого входить кокарбоксилаза (тіаміндифосфат) - коферментна форма вітаміну: To improve the trophism of the heart muscle, the patient is prescribed cardonate, which includes cocarboxylase (thiamine diphosphate) - a coenzyme form of the vitamin:

В12 B12

В10 B10

В1 B1

В5 B5

В2 B2

111 / 200
Виділення гормонів кори наднирників регулюється АКТГ аденогіпофізу. Які гормони виділяються наднирниками при дії останнього? The secretion of hormones by the adrenal cortex is regulated by the ACTH of the adenohypophysis. What hormones are secreted by the adrenal glands under the action of the latter?

Глюкокортикоїди Glucocorticoids

Катехоламіни Catecholamines

Простагландини Prostaglandins

Андрогени Androgens

Мінералокортикоїди Mineralocorticoids

112 / 200
До гінеколога звернулася жінка 28ми років з приводу безпліддя. При обстеженні знайдено: недорозвинені яєчники і матка, нерегулярний менструальний цикл. При дослідженні статевого хроматину у більшості соматичних клітин виявлено 2 тільця Бара. Яка хромосомна хвороба найбільш імовірна у жінки? A 28-year-old woman consulted a gynecologist about infertility. The examination revealed: underdeveloped ovaries and uterus, an irregular menstrual cycle. When examining the sex chromatin in most somatic cells, 2 Barr bodies. What chromosomal disease is most likely in a woman?

Синдром Патау Patau syndrome

Синдром трипло-Х Triple X Syndrome

Синдром Едвардса Edwards Syndrome

Синдром Шерешевського-Тернера Shereshevsky-Turner syndrome

Синдром Клайнфельтера Klinefelter syndrome

113 / 200
У пацієнтки з артеріальною гіпертензією виник гіпертензивний криз, який ускладнився набряком легень. Який сечогінний препарат необхідно застосувати у комплексній терапії даної хворої? A hypertensive crisis occurred in a patient with arterial hypertension, which was complicated by pulmonary edema. What diuretic should be used in the complex therapy of this patient?

Верошпірон Veroshpiron

Тріамтерен Triamterene

Еуфілін Euphilin

Амілорид Amiloride

Фуросемід Furosemide

114 / 200
У 2-річної дівчинки була діагностована важка форма міастенії. Який з перерахованих засобів показаний для лікуванні цього захворювання? A 2-year-old girl was diagnosed with a severe form of myasthenia gravis. Which of the listed means is indicated for the treatment of this disease?

Галантаміну гідробромід Galantamine hydrobromide

Допамін Dopamine

Норадреналін Noradrenaline

Цитітон Cititon

Скополамін Scopolamine

115 / 200
Сімейний лікар призначив хворому 53-х років для лікування хронічної серцевої недостатності дигоксин. Через 1 місяць виявив симптоми глікозидної інтоксикації. Який препарат необхідно призначити для усунення інтоксикації? The family doctor prescribed digoxin to a 53-year-old patient for the treatment of chronic heart failure. After 1 month, he showed symptoms of glycoside intoxication. What drug should be prescribed to eliminate intoxication?

Ціанокобаламін Cyanocobalamin

Морфіну гідрохлорид Morphine hydrochloride

Аміназин Aminazine

Унітіол Unithiol

Мезатон Mesaton

116 / 200
При огляді 6-місячної дитини лікар виявив незакрите заднє тім’ячко. При нормальному розвитку дитини воно закривається до: During an examination of a 6-month-old child, the doctor found an open posterior parietal lobe. With normal child development, it closes until:

6-ти місяців 6 months

Кінця другого року життя End of second year of life

Народження Birth

3-х місяців 3 months

Кінця першого року життя End of first year of life

117 / 200
У хлопчика водянка яєчка (накопичування рідини між оболонками яєчка). Яка саме оболонка яєчка вміщує цю рідину? A boy has hydrocele (accumulation of fluid between the scrotum). Which scrotum contains this fluid?

Білкова Bilkova

Піхвова Vagina

Зовнішня сім’яна External seed

Внутрішня сім’яна Internal seed

М’ясиста Fleshy

118 / 200
Хворому на хронічний гастрит зроблена внутрішньошлункова рН-метрія, за допомогою якої встановлено зменшення кислотності шлункового соку. Функція яких клітин знижена? A patient with chronic gastritis underwent intragastric pH-metry, with the help of which a decrease in the acidity of gastric juice was established. The function of which cells is reduced?

Парієтальні екзокриноцити Parietal exocrinocytes

Додаткові клітини Additional Cells

Шийкові клітини Cervical cells

Ендокриноцити Endocrinocytes

Головні екзокриноцити Main exocrinocytes

119 / 200
З метою визначення максимальної секреції хлористоводневої кислоти шлункового соку пацієнту 42-х років ввели розчин гістаміну. Це призвело до збільшення секреції підшлунковою залозою такого компоненту соку: In order to determine the maximum secretion of hydrochloric acid of gastric juice, a 42-year-old patient was injected with a solution of histamine. This led to an increase in the secretion of the following component of the juice by the pancreas:

Трипсиноген Trypsinogen

Слиз Slime

Ліпаза Lipase

Бікарбонати Bicarbonates

Амілаза Amylase

120 / 200
У хворого з облітеруючим атеросклерозом проведено десимпатизацію стегнової артерії в ділянці стегнового трикутника. Який вид артеріальної гіперемії виник внаслідок операції'? In a patient with obliterating atherosclerosis, the femoral artery was desympathized in the area of the femoral triangle. What type of arterial hyperemia occurred as a result of the operation?

Нейропаралітична Neuroparalytic

Реактивна Reactive

Метаболічна Metabolic

Робоча Working

Нейротонічна Neurotic

121 / 200
Хворий потрапив до лікарні після іонізуючого опромінення зі скаргами на блювання, анорексію, біль в різних ділянках живота, наявність крові у калі, підвищення температури тіла, млявість. Для якої форми гострої променевої хвороби характерна клінічна картина? The patient was admitted to the hospital after ionizing radiation with complaints of vomiting, anorexia, pain in various parts of the abdomen, the presence of blood in the stool, increased body temperature, lethargy. For which What is the characteristic clinical picture of forms of acute radiation sickness?

Змішана Mixed

Кістковомозкова Bone marrow

Кишкова Intestinal

Церебральна Cerebral

Токсемічна Toxemic

122 / 200
В нефрологічній клініці у юнака 19-ти років була виявлена підвищена кількість калію у вторинній сечі. Підвищення секреції якого гормону, імовірно могло викликати такі зміни? In the nephrology clinic, a 19-year-old man was found to have an increased amount of potassium in the secondary urine. An increase in the secretion of which hormone could probably cause such changes?

Адреналін Adrenaline

Альдостерон Aldosterone

Тестостерон Testosterone

Окситоцин Oxytocin

Глюкагон Glucagon

123 / 200
У хворого на інсульт спостерігається порушення заднього відділу задньої ніжки внутрішньої капсули. Які з провідних шляхів там проходять? A stroke patient has a violation of the posterior part of the posterior leg of the internal capsule. Which of the leading pathways pass there?

Radiatio acustica et radiatio optica Radiatio acustica et radiatio optica

Tr. Parietopontinae Tr. Parietopontinae

Tr. Corticospinalis Tr. Corticospinalis

Tr Occipitopontinae Tr Occipitopontinae

Tr. Corticonuclearis Tr. Corticonuclearis

124 / 200
У хворого 40-а років в результаті щелепно-лицьової травми порушилася функція під’язикової і підщелепної залоз зліва - залози почали секретувати невелику кількість густої слини. Функція якого нерва порушена? In a 40-year-old patient, as a result of a maxillofacial injury, the function of the hypoglossal and submandibular glands on the left was disturbed - the glands began to secrete a small amount of thick saliva. The function of which nerve violated?

Лицевий Face

Під’язиковий Sublingual

Язикоглотковий Lingopharyngeal

Трійчастий Triple

Блукаючий Wandering

125 / 200
На аутопсії померлого, який тривало страждав на гіпертонічну хворобу, патологоанатом виявив, що нирки різко зменшені у розмірах, щільні, поверхня їх рівномірно дрібнозерниста, на розрізі - паренхіма, особливо кіркова речовина, рівномірно виточені. Він дійшов висновку, що це: At the autopsy of the deceased, who suffered from hypertension for a long time, the pathologist found that the kidneys were sharply reduced in size, dense, their surface was uniformly fine-grained, the parenchyma was especially the cortical substance, uniformly refined. He concluded that it:

Атеросклеротично зморщена нирка Atherosclerotic shrunken kidney

Пієлонефритично зморщена нирка Pyelonephritic shrunken kidney

Амілоїдно-зморщена нирка Amyloid shrunken kidney

Артеріолосклеротично зморщена нирка Arteriolosclerotic shrunken kidney

- -

126 / 200
У хворого в порожнині рота спостерігається повна атрофія сосочків язика, язик червоний, 'лакований', вкритий виразками. В крові: лейкопенія, зсув формули вправо, КП-1,3, мегалоцити, тільця Жолі. Для якої патології це характерно? The patient has a complete atrophy of the papillae of the tongue in the oral cavity, the tongue is red, 'lacquered', covered with ulcers. In the blood: leukopenia, shift of the formula to the right, KP-1 ,3, megalocytes, Joly bodies. What pathology is this characteristic of?

Залізодефіцитна анемія Iron deficiency anemia

B12 -дефіцитна анемія B12 deficiency anemia

Гемолітична анемія Hemolytic anemia

Залізорефрактерна анемія Iron refractory anemia

Агранулоцитоз Agranulocytosis

127 / 200
При вивченні препарату трубчастого органу встановлено, що його середня оболонка утворена гіаліновою хрящовою тканиною, яка формує незамкнені кільця. Який це орган? When studying a preparation of a tubular organ, it was established that its middle shell is formed by hyaline cartilaginous tissue that forms open rings. What kind of organ is this?

Малі бронхи Small bronchi

Головні бронхи Main bronchi

Трахея Trachea

Великі бронхи Large bronchi

Термінальні бронхіоли Terminal bronchioles

128 / 200
В тканині видаленої слинної залози знайдено: дифузний склероз, прошарки сполучної тканини містять змішаноклі-тинний інфільтрат, часточки залози атрофовані, протоки розширені. Крім того, тканина залози містить кістозну порожнину великих розмірів, внутрішня поверхня її вкрита сплощеним двошаровим епітелієм, просвіт заповнений рідиною, оточуюча тканина склерозована. Діагностуйте захворювання: In the tissue of the removed salivary gland, the following were found: diffuse sclerosis, the layers of connective tissue contain a mixed cellular infiltrate, the lobes of the gland are atrophied, the ducts are dilated. In addition, the tissue of the gland contains cystic a large cavity, its inner surface is covered with a flattened two-layered epithelium, the lumen is filled with liquid, the surrounding tissue is sclerosed. Diagnose the disease:

Плеоморфна аденома Pleomorphic adenoma

Ретенційна кіста Retention cyst

Сіалолітіаз Sialolithiasis

Мономорфна аденома Monomorphic adenoma

Гострий сіалоаденіт Acute sialoadenitis

129 / 200
При втручанні з метою лікування вивиху нижньої щелепи лікар повинен пам’ятати про м’яз, який при скороченні відтягує назовні капсулу і суглобовий диск скронево- нижньощелепного суглоба. Який це м’яз? When intervening to treat a dislocation of the lower jaw, the doctor must remember the muscle that, when contracted, pulls the capsule and articular disc of the temporomandibular joint outward. What is it a muscle?

M. pterygoideus medialis M. pterygoideus medialis

M. pterygoideus lateralis M. pterygoideus lateralis

M. mylohyoideus M. mylohyoideus

M. masseter M. masseter

M. temporalis M. temporalis

130 / 200
Зірчастий ретикулум і зовнішній шар клітин емалевого органу піддаються інволюції і після завершення амелогенезу разом з апікальними частинами амелобластів формують: The stellate reticulum and the outer layer of cells of the enamel organ undergo involution and after completion of amelogenesis together with the apical parts of ameloblasts form:

- -

Кутикулу зуба Tooth cuticle

Муцинову плівку Mucin film

Зубний наліт (бляшки) Plaque

Пелікулу зуба Tooth pellicle

131 / 200
У 4-річного хлопчика з’явились горбисті, щільні, підшкірно розташовані утворення в ділянці кутів та вздовж гілок двох нижніх щелеп, які зумовили деформацію обличчя. При гістологічному дослідженні біоптату виявлено: в міжбалковому просторі розростання васкуляризованої сполучної тканини, довкола судин маси ацидофільного матеріалу, багатоядерні гігантські клітини, окремі примітивні кісткові балочки. Діагностуйте захворювання: In a 4-year-old boy, lumpy, dense, subcutaneous formations appeared in the area of the corners and along the branches of the two lower jaws, which caused the deformation of the face. During histological examination of the biopsy revealed: in the inter-beam space, the growth of vascularized connective tissue, around the vessels, masses of acidophilic material, multinucleated giant cells, individual primitive bone beams. Diagnose the disease:

Хронічний фіброзний періостит Chronic fibrous periostitis

Еозинофільна гранульома Eosinophilic granuloma

Периферійна гігантоклітинна гранульома Peripheral giant cell granuloma

Фіброзна дисплазія Fibrous dysplasia

Херувізм Cherubism

132 / 200
При дослідженні бронхобіоптату встановлено атрофію слизової оболонки, кістозне перетворення залоз, осередкову метаплазію покривного призматичного епітелію в багатошаровий плоский, збільшення числа келихоподібних клітин, місцями у стінці бронха та особливо у слизовій оболонці різко виражена клітинна запальна інфільтрація і розростання грануляційної тканини, яка вибухає у просвіт бронха у вигляді поліпа. Який найбільш імовірний діагноз? During the examination of the bronchobioptate, atrophy of the mucous membrane, cystic transformation of the glands, focal metaplasia of the covering prismatic epithelium into a multilayered flat one, an increase in the number of goblet cells, in places in the wall of the bronchus and especially in the mucous membrane has a marked cellular inflammatory infiltration and the growth of granulation tissue, which explodes into the lumen of the bronchus in the form of a polyp. What is the most likely diagnosis?

Проміжна пневмонія Intermediate pneumonia

Гострий бронхіт Acute bronchitis

Гостра пневмонія Acute pneumonia

Хронічний бронхіт Chronic bronchitis

Бронхопневмонія Bronchopneumonia

133 / 200
При остеолатеризмі зменшується міцність колагену, що зумовлена помітним зменшенням утворення поперечних зшивок у колагенових фібрилах. Причиною цього явища є зниження активності такого ферменту: With osteolaterism, the strength of collagen decreases, which is due to a noticeable decrease in the formation of cross-links in collagen fibrils. The reason for this phenomenon is a decrease in the activity of such an enzyme:

Лізилоксидаза Lysyl oxidase

Лізилгідроксилаза Lysylhydroxylase

Пролілгідроксилаза Prolylhydroxylase

Колагеназа Collagenase

Моноаміноксидаза Monoamine oxidase

134 / 200
Під час мікроскопічного дослідження в залозі дитини 10-ти років були визначені тільки серозні кінцеві відділи, міжчасточкові протоки вистелені двошаровим чи багатошаровим епітелієм. Визначте залозу: During a microscopic examination of the gland of a 10-year-old child, only serous end sections were identified, the interlobular ducts were lined with two-layered or multilayered epithelium. Identify the gland:

Під’язикова Sublingual

Печінка Liver

Привушна Privushna

Підшлункова Pancreatic

Підщелепна Submaxillary

135 / 200
Хвороба Хартнепа зумовлена точковою мутацією лише одного гена, наслідком чого є порушення всмоктування амінокислоти триптофану в кишечнику та реабсорбції її в ниркових канальцях. Це призводить до одночасних розладів у травній і сечовидільній системах. Яке генетичне явище спостерігається в цьому випадку? Hartnap's disease is caused by a point mutation of only one gene, the result of which is a violation of the absorption of the amino acid tryptophan in the intestine and its reabsorption in the renal tubules. This leads to simultaneous disorders in the digestive and urinary systems. What genetic phenomenon is observed in this case?

Комплементарна взаємодія Complementary interaction

Неповне домінування Incomplete dominance

Полімерія Polymeria

Плейотропія Pleiotropy

Кодомінування Codominance

136 / 200
Відомо, що частина диоксиду вуглецю використовується в організмі в біосинтезі жирних кислот, сечовини, глюконеогенезі тощо. Який вітамін утворює СO2-транспортуючу форму для цих реакцій? It is known that part of carbon dioxide is used in the body in the biosynthesis of fatty acids, urea, gluconeogenesis, etc. What vitamin forms the CO2-transporting form for these reactions?

Нікотинамід Nicotinamide

Рибофлавін Riboflavin

Тимін Thymin

Ретинол Retinol

Біотин Biotin

137 / 200
У хворого, який страждає на вугрі та запальні зміни шкіри обличчя, при мікроскопії матеріалу з осередків ураження виявлені живі членистоногі розміром 0,2-0,5 мм. Вони мають витягнуту червоподібну форму, чотири пари коротких кінцівок, розміщених у середній частині тіла. Виявлені членистоногі викликають таке захворювання: In a patient suffering from acne and inflammatory changes in the skin of the face, live arthropods with a size of 0.2-0.5 mm were found during microscopy of the material from the lesions. They have an elongated worm-like shape, four pairs of short limbs located in the middle part of the body. The identified arthropods cause the following disease:

Демодекоз Demodecosis

Педикульоз Pediculosis

Шкірний міаз Cutaneous myiasis

Фтиріоз Phthyriosis

Короста Scabies

138 / 200
У хворого 35-ти років травма голови викликала втрату тактильних і температурних відчуттів. Яку звивину було пошкоджено внаслідок травми? In a 35-year-old patient, a head injury caused loss of tactile and temperature sensations. Which gyrus was damaged as a result of the injury?

Передцентральна Precentral

Зацентральна Centered

Кутова Angular

Поясна Belt

Надкрайова Ultimate

139 / 200
Сечокам’яна хвороба ускладнилася виходом конкременту з нирки. На якому рівні сечоводу, найімовірніше, він може зупинитися? The urolithiasis was complicated by the release of a stone from the kidney. At what level of the ureter is it most likely to stop?

В середній черевній частині In the middle abdomen

На 5 см вище тазової частини 5 cm above the pelvis

У нирковій мисці In the renal pelvis

На межі черевної та тазової частин On the border of the abdominal and pelvic parts

На 2 см вище впадіння в сечовий міхур 2 cm above the confluence with the bladder

140 / 200
В гістологічному препараті нижньої щелепи ембріону виявляється зубний зачаток, в якому зубний сосочок утворений дрібними зірчастими базофільно забарвленими клітинами. Яка тканина утворює цю частину зубного зачатку? In the histological preparation of the lower jaw of the embryo, a tooth bud is revealed, in which the dental papilla is formed by small stellate basophil-stained cells. What tissue forms this part of the tooth bud?

Ретикулярна Reticular

Епітеліальна Epithelial

Кісткова Bone

Мезенхіма Mesenchyme

Хрящова Khryaschova

141 / 200
При огляді ротової порожнини на вестибулярній поверхні нижнього різця зліва виявлене утворення грибоподібної форми рожевого кольору до 2 см, яке широкою ніжкою фіксоване до надальвео-лярної тканини. Під час гістологічного дослідження виявлено розгалужені судини капілярного типу з судинними бруньками, ділянками крововиливів та осередки гемосидерозу Який найбільш імовірний діагноз? When examining the oral cavity on the vestibular surface of the lower incisor on the left, a mushroom-shaped formation of pink color up to 2 cm was found, which was fixed to the supraalveolar tissue with a wide stalk. During histological examination the study revealed branched vessels of the capillary type with vascular buds, areas of hemorrhages and foci of hemosiderosis. What is the most likely diagnosis?

Фіброзний епуліс Fibrous epulis

Гігантоклітинний епуліс Giant cell epulis

Фіброматоз ясен Gingival fibromatosis

Кавернозна гемангіома Cavernous hemangioma

Ангіоматозний епуліс Angiomatous epulis

142 / 200
Хворому з важким перебігом ревматоїдного артриту для постійного прийому був призначений преднізолон. Який режим прийому є оптимальним з урахуванням хронофармакологічних особливостей дії преднізолону та фізіологічних коливань секреції кортикостероїдів в організмі? A patient with a severe course of rheumatoid arthritis was prescribed prednisone for continuous administration. What regimen is optimal taking into account the chronopharmacological features of the action of prednisone and the physiological fluctuations of the secretion of corticosteroids in the body?

1/3 дози тричі на день 1/3 dose three times a day

Усі наведені схеми рівноцінні All given schemes are equivalent

2/3 добової дози вранці, 1/3 дози вдень 2/3 daily dose in the morning, 1/3 dose in the afternoon

Вся доза вранці Full dose in the morning

Вся доза ввечері All dose in the evening

143 / 200
Пацієнт 68-ми років звернувся до кардіолога із скаргами на підвищення артеріального тиску, біль в ділянці серця, відчуття перебоїв у роботі серця. Призначте препарат з групи β1 -адреноблокаторів для лікування даної патології: A 68-year-old patient turned to a cardiologist with complaints of increased blood pressure, pain in the heart area, a feeling of heart failure. Prescribe a drug from the group of β1-adrenoblockers for the treatment of this pathology:

Ноотропіл Nootropil

Метопролол Metoprolol

Бензилпеніцилін Benzylpenicillin

Морфіну гідрохлорид Morphine hydrochloride

Фенотерол Fenoterol

144 / 200
Людина згинає та розгинає передпліччя без навантаження, спираючись ліктем на стіл. Який вид м’язового скорочення має місце у m. biceps brachii? A person bends and extends the forearm without load, resting his elbow on the table. What type of muscle contraction occurs in m. biceps brachii?

Ізометричне Isometric

Ауксотонічне Auxotonic

Ізотонічне Isotonic

Зубчастий тетанус Serrated tetanus

Гладкий тетанус Smooth tetanus

145 / 200
У жінки, що тривало приймала антибіотики з приводу кишкової інфекції, розвинулося ускладнення з боку слизової порожнини рота у вигляді запального процесу і білого нальоту, у якому при бактеріологічному дослідженні були виявлені дріжджеподібні грибки Candida albicans. Який з перерахованих препаратів показаний для лікування цього ускладнення? A woman who had been taking antibiotics for a long time due to an intestinal infection developed complications from the mucous membrane of the mouth in the form of an inflammatory process and white plaque, in which bacteriological examination showed yeast-like fungi Candida albicans were detected. Which of the listed drugs is indicated for the treatment of this complication?

Флуконазол Fluconazole

Тетрациклін Tetracycline

Фуразолідон Furazolidone

Поліміксин Polymixin

Бісептол Biseptol

146 / 200
Мембранний потенціал спокою клітини змінився з -85 мВ до -90 м. Причиною цього може бути активація таких каналів мембрани клітини: The resting membrane potential of the cell has changed from -85 mV to -90 mV. The reason for this may be the activation of the following cell membrane channels:

Натрієві Sodium

Калієві та кальцієві Potassium and calcium

Кальцієві Calcium

Калієві Potassium

Калієві та натрієві Potassium and sodium

147 / 200
Під час розтину тіла виявлено: легені щільні, коричневого кольору за рахунок відкладання ендогенного пігменту. Відомо, що за життя у хворого мав місце хронічний венозний застій в малому колі кровообігу. Який патологічний процес викликав таку картину? During the autopsy, the lungs were found to be dense, brown in color due to the deposition of endogenous pigment. It is known that the patient had chronic venous stasis in the small blood circulation during his life . What pathological process caused such a picture?

Кальциноз Calcinosis

Меланоз Melanosis

Гемосидероз Hemosiderosis

Жовтяниця Jaundice

Порфірія Porphyria

148 / 200
Організми мають ядро, оточене ядерною мембраною. Генетичний матеріал зосереджений переважно в хромосомах, які складаються з ниток ДНК і білкових молекул. Діляться ці клітини мітотично. Це: Organisms have a nucleus surrounded by a nuclear membrane. Genetic material is concentrated mainly in chromosomes, which consist of DNA strands and protein molecules. These cells divide mitotically. This is:

Еукаріоти Eukaryotes

Бактерії Bacteria

Віруси Viruses

Прокаріоти Prokaryotes

Бактеріофаги Bacteriophages

149 / 200
Хвора звернулася зі скаргами на припухлість в передній ділянці шиї, схуднення, випуклість очей, тахікардію, неврівноваженість. Яка залоза уражена? The patient complained of swelling in the front of the neck, weight loss, bulging eyes, tachycardia, imbalance. Which gland is affected?

Гіпофіз Pituitary

Щитоподібна залоза Thyroid

Епіфіз Pineal gland

Прищитоподібні залози Parathyroid glands

Тимус Thymus

150 / 200
Під час аускультації хворого попросили глибоко дихати. Після 10 дихальних рухів хворий знепритомнів, що пов’язано з: During auscultation, the patient was asked to breathe deeply. After 10 breaths, the patient fainted, which is due to:

Еритроцитозом Erythrocytosis

Зменшенням кисневої ємності крові Decreased blood oxygen capacity

Еритропенією Erytropenia

Респіраторним ацидозом Respiratory acidosis

Респіраторним алкалозом Respiratory alkalosis

151 / 200
Хворому на виразкову хворобу шлунка 45-ти років призначено препарат в основі дії якого є антимікробний ефект. Який це препарат? A 45-year-old stomach ulcer patient is prescribed a drug based on an antimicrobial effect. What is this drug?

Бісакодил Bisacodyl

Дитилін Dithyline

Фталазол Phthalazole

Метронідазол Metronidazole

Омепразол Omeprazole

152 / 200
Одна з тканин зуба постійно відкладається в області верхівки кореня, що обумовлює його видовження. Цей процес компенсує стирання коронки і забезпечує постійність загальної довжини зуба. Яка тканина зуба є репаративно активною у постнатальному періоді? One of the tissues of the tooth is constantly deposited in the region of the apex of the root, which causes its elongation. This process compensates for the wear of the crown and ensures the constancy of the total length of the tooth. Which tissue of the tooth is reparative active in the postnatal period?

Цемент Cement

Щільна сполучна Dense connective

Емаль Enamel

Пухка сполучна Loose connector

Кісткова Bone

153 / 200
Чоловіка 29-ти років непокоїть різкий біль шкіри обличчя. Який нерв уражений? A 29-year-old man is troubled by a sharp pain in the skin of his face. Which nerve is affected?

Лицевий Face

Блукаючий Wandering

Язикоглотковий Lingopharyngeal

Трійчастий Triple

Окоруховий Okoruhovy

154 / 200
У людей, які проживають на великих висотах над рівнем моря, відмічається збільшення кількості еритроцитів, що спричиняється: In people who live at high altitudes above sea level, an increase in the number of red blood cells is noted, which is caused by:

Впливом гіпоксії на розпад еритроцитів Influence of hypoxia on the breakdown of erythrocytes

Гальмівним впливом гіпоксії на кістковий мозок Inhibitory effect of hypoxia on bone marrow

Підвищеним споживанням тканинами кисню Increased tissue oxygen consumption

Посиленим виділенням нирками реніну Increased excretion of renin by the kidneys

Посиленим утворенням нирками еритропоетину Increased production of erythropoietin by the kidneys

155 / 200
У бактеріологічній лабораторії проводилася експертиза шкур тварин за допомогою імунної реакції преципітації за Асколі. Позитивний результат цієї реакції свідчить про наявність: In the bacteriological laboratory, an examination of animal skins was carried out using the immunoprecipitation reaction according to Ascoli. A positive result of this reaction indicates the presence of:

Збудника чуми The causative agent of the plague

Токсину анаеробної інфекції Anaerobic infection toxin

Поверхневого антигену ієрсиній Yersinia surface antigen

Збудника бруцельозу The causative agent of brucellosis

Антигенів збудника сибірки Anthrax antigens

156 / 200
Після вживання м’ясної консерви у школяра з’явилися неврологічні симптоми. Був поставлений діагноз: ботулізм. Які екстрені методи лікування необхідно використати? After eating canned meat, a schoolboy developed neurological symptoms. The diagnosis was botulism. What emergency treatment methods should be used?

Введення антиботулінічної вакцини Introduction of anti-botulinum vaccine

Введення антиботулінічної сироватки Introduction of anti-botulinum serum

Призначення проносних засобів Prescribing laxatives

Введення сульфаніламідних препаратів Introduction of sulfonamide drugs

Введення антибіотиків Introduction of antibiotics

157 / 200
У хворого запальний процес у крилопіднебінній ямці. Інфекція поширилась в носову порожнину. Через яке анатомічне утворення розповсюдилась інфекція? The patient has an inflammatory process in the pterygopalatine fossa. The infection spread to the nasal cavity. Through which anatomical structure did the infection spread?

Foramen sphenopalatinum Foramen sphenopalatinum

Foramen rotundum Foramen rotundum

Canalis palatinus minor Canalis palatinus minor

Canalis ptherygoideus Canalis ptherygoideus

Canalis palatinus major Canalis palatinus major

158 / 200
У пацієнта 32-х років має місце гіповітаміноз B2. Причиною виникнення специфічних симптомів (ураження епітелію, слизових, шкіри, рогівки ока) найбільш імовірно є дефіцит: A 32-year-old patient has hypovitaminosis B2. The cause of specific symptoms (epithelial, mucous, skin, corneal lesions) is most likely a deficiency:

Цитохромоксидази Cytochrome oxidases

Цитохрому а1 Cytochrome a1

Цитохрому с Cytochrome c

Флавінових коферментів Flavin coenzymes

Цитохрому в Cytochrome in

159 / 200
Жінці, яка скаржиться на постійне відчуття страху, тривоги, поставлено діагноз неврозу та призначено препарат з анксіолітичною дією. Який це препарат? A woman who complains of a constant feeling of fear, anxiety, was diagnosed with neurosis and prescribed a drug with an anxiolytic effect. What is this drug?

Діазепам Diazepam

Аміназин Aminazine

Настойка женьшеню Ginseng Tincture

Пірацетам Piracetam

Кофеїн-бензоат натрію Caffeine sodium benzoate

160 / 200
У пробірку, що містить 0,3% розчин NaCl, додали краплю крові. Що відбудеться з еритроцитами? A drop of blood was added to a test tube containing 0.3% NaCl solution. What will happen to the erythrocytes?

Змін не буде There will be no changes

Біологічний гемоліз Biological hemolysis

Осмотичний гемоліз Osmotic hemolysis

Зморшкування Wrinkle

Механічний гемоліз Mechanical hemolysis

161 / 200
Після перенесеної травми голови у хворого бувають напади інтенсивного болю в ділянці обличчя і судоми жувальних м’язів. Який нерв найімовірніше травмований? After a head injury, the patient has attacks of intense pain in the face and spasms of the masticatory muscles. Which nerve is most likely injured?

N. oculomotorius N. oculomotorius

N. trigeminus N. trigeminus

N. olphactorius N. olphactorius

N. facialis N. facialis

N. abducens N. abducens

162 / 200
Вивчається родовід сім’ї, в якій спостерігаються надмірно великі зуби. Ця ознака трапляється в усіх поколіннях тільки у чоловіків і успадковується від батька до сина. Визначте тип успадкування : The genealogy of a family in which excessively large teeth are observed. This trait occurs in all generations only in men and is inherited from father to son. Determine the type of inheritance:

Зчеплений з Х-хромосомою рецесивний X-linked recessive

Автосомно-рецесивний Autosomal recessive

Автосомно-домінантний Autosomal dominant

Зчеплений з Х-хромосомою домінантний X-linked dominant

Зчеплений з Y-хромосомою Linked to Y-chromosome

163 / 200
У постраждалого перелом тіла нижньої щелепи. Кровотеча з рани. Яка артерія ушкоджена? The victim has a fracture of the body of the lower jaw. Bleeding from the wound. Which artery is damaged?

A. lingualis A. lingualis

A. facialis A. facialis

A. carotis externa A. carotis externa

A. maxillaris A. maxillaris

A. alveolaris inferior A. alveolaris inferior

164 / 200
На розтині тіла 68-річної жінки, що померла від гострої серцевої недостатності, у задній стінці лівого шлуночка серця виявлена ділянка неправильної форми, розмірами 6,5х4,5 см, в’яла, блідо-жовтуватого кольору, оточена зоною гіперемії. Як розцінив патологоанатом виявлені зміни? At the autopsy of a 68-year-old woman who died of acute heart failure, an irregularly shaped area measuring 6.5x4.5 cm was found in the back wall of the left ventricle of the heart , flabby, pale yellowish in color, surrounded by a zone of hyperemia. How did the pathologist assess the changes detected?

Аневризма серця Heart aneurysm

Післяінфарктний кардіосклероз Postinfarction cardiosclerosis

Дифузний кардіосклероз Diffuse cardiosclerosis

Гострий інфаркт міокарда Acute myocardial infarction

Вогнищевий міокардит Focal myocarditis

165 / 200
На розтині тіла чоловіка 47-ми років, що помер від легеневої кровотечі, у 2 сегменті правої легені виявлена порожнина округлої форми з нерівними краями розміром 5,5 см; внутрішня поверхня її вкрита щільнуватими жовтуватими масами, що переходять у тканину легені. При гістологічному дослідженні - внутрішній шар складається з розплавлених казеозних мас, некротизованої тканини легені. Який процес розвинувся в легені? At the autopsy of a 47-year-old man who died of pulmonary hemorrhage, a 5.5 cm round cavity with uneven edges was found in the 2nd segment of the right lung; its inner surface is covered with dense yellowish masses that pass into the lung tissue. In histological examination, the inner layer consists of melted caseous masses, necrotized lung tissue. What process developed in the lung?

Гострий кавернозний туберкульоз Acute cavernous tuberculosis

Рак легені, що розпадається Decomposing lung cancer

Гострий абсцес Acute abscess

Хронічний абсцес Chronic abscess

Хронічний кавернозний туберкульоз Chronic cavernous tuberculosis

166 / 200
Екзотоксин дифтерійної палички обробили 0,3-0,4% формаліном і витримали 30 днів у термостаті при температурі 40o Який препарат був отриманий у результаті проведених маніпуляцій? The diphtheria bacillus exotoxin was treated with 0.3-0.4% formalin and kept for 30 days in a thermostat at a temperature of 40o. What preparation was obtained as a result of the manipulations?

Лікувальна сироватка Healing Serum

Анатоксин Anatoxin

Діагностикум Diagnostics

Антитоксин Antitoxin

Діагностична сироватка Diagnostic serum

167 / 200
В економічно розвинених країнах поширеним захворюванням є карієс зубів. Це захворювання уражає більш 95% населення. Що відіграє головну роль у демінералізації твердих тканин зуба при карієсі? Dental caries is a common disease in economically developed countries. This disease affects more than 95% of the population. What plays the main role in the demineralization of hard tooth tissues during caries?

Екстремальні впливи на організм Extreme effects on the body

Недостатність вітаміну C Vitamin C deficiency

Розлад регуляції метаболізму Metabolism regulation disorder

Органічні кислоти Organic acids

Неповноцінне харчування Malnutrition

168 / 200
При вивченні гістологічного препарату пульпи зуба було відмічено, що в сполучній тканині переважають пучки колагенових волокон, шар дентинобластів тонкий, проміжний шар виражений слабо. В якій ділянці зуба пульпа має такі особливості? When studying the histological preparation of the tooth pulp, it was noted that bundles of collagen fibers predominate in the connective tissue, the layer of dentinoblasts is thin, and the intermediate layer is weakly expressed. In which part of the tooth does the pulp have such features?

Коренева пульпа Root pulp

Шар Вейля Weil Layer

Коронкова пульпа Crown pulp

Центральний шар пульпи Central pulp layer

Периферичний шар пульпи Peripheral pulp layer

169 / 200
При огляді ротової порожнини стоматолог виявив каріозну порожнину на поверхні коронки другого малого кутнього зуба, яка межує з першим великим кутнім зубом. Назвіть пошкоджену поверхню коронки: When examining the oral cavity, the dentist found a carious cavity on the surface of the crown of the second small canine tooth, which borders the first large canine tooth. Name the damaged surface of the crown:

Facies distalis Facies distalis

Facies mesialis Facies mesialis

Facies vestibularis Facies vestibularis

Facies lingualis Facies lingualis

Facies occlusalis Facies occlusalis

170 / 200
У собаки вироблений умовний слиновидільний рефлекс на вмикання світла. Вмикання дзвоника під час виконання даного рефлексу призведе до розвитку наступного виду гальмування: The dog has developed a conditional salivary reflex to turn on the light. Turning on the bell during the execution of this reflex will lead to the development of the following type of inhibition:

Позамежне Foreign

Зовнішнє External

Диференціювання Differentiation

Згасання Extinction

Умовне гальмування Conditional braking

171 / 200
Чоловік звернувся до стоматолога зі скаргою, що в нього нижня щелепа не рухається назад. Встановлено, що в нього пошкоджений такий м’яз : A man went to the dentist with a complaint that his lower jaw does not move back. It was established that the following muscle was damaged in him :

Двочеревцевий Bigastric

Скроневий Temporal

Присередній крилоподібний Medium pterygium

Бічний крилоподібний Lateral pterygoid

Жувальний Chewing

172 / 200
У пацієнта внаслідок запального процесу виникло надмірне збудження вушно-скроневого нерва. При цьому привушною слинною залозою буде виділятися: As a result of the inflammatory process, the patient has excessive excitation of the ototemporal nerve. At the same time, the parotid salivary gland will secrete:

Виділення слини припиниться Salivation will stop

Мала кількість рідкої слини Small amount of liquid saliva

Мала кількість в’язкої слини Small amount of viscous saliva

Велика кількість в’язкої слини Lots of viscous saliva

Велика кількість рідкої слини A large amount of liquid saliva

173 / 200
Хворому 55-ти років для комплексного лікування легеневої форми туберкульозу призначений протитуберкульозний засіб. Який з вказаних лікарських засобів проявляє свою антимікробну активність виключно по відношенню до мікобактерій туберкульозу? A 55-year-old patient is prescribed an anti-tuberculosis drug for complex treatment of pulmonary tuberculosis. Which of the listed drugs shows its antimicrobial activity exclusively against tuberculosis mycobacteria?

Стрептоміцину сульфат Streptomycin sulfate

Канаміцину сульфат Kanamycin sulfate

Циклосерин Cycloserine

Гатіфлоксацин Gatifloxacin

Ізоніазид Isoniazid

174 / 200
У дитини 9-ти років швидко підвищилась температура тіла до 39°С, з’явився біль у горлі. Об’єктивно: зів та мигдалики яскраво-червоного кольору. Язик набряклий малиново- червоного кольору зі збільшеними грибоподібними сосочками. Шкіра тіла та обличчя, крім носо-губного трикутника, вкрита густими червоними плямами величиною з макове зерно. Підщелепні лімфатичні вузли при пальпації болісні. Яке захворювання у дитини? A 9-year-old child's body temperature quickly rose to 39°С, a sore throat appeared. Objectively: the pharynx and tonsils are bright red . The tongue is swollen crimson-red in color with enlarged mushroom-like papillae. The skin of the body and face, except for the nasolabial triangle, is covered with thick red spots the size of a poppy seed. Submaxillary lymph nodes are painful when palpated. What disease does the child have?

Поліомієліт Polio

Дифтерія Diphtheria

Скарлатина Scarlatina

Кір Measles

Менінгококовий назофарингіт Meningococcal nasopharyngitis

175 / 200
Хвора 20-ти років звернулась до лікаря зі скаргами на кровоточивість та біль у яснах, що з’явилися через 2 дні після прийому сульфадимезину. Об’єктивно: ясенні сосочки та ясенний край гіперемовані, набряклі, кровоточать при незначному подразненні. Яка патологія розвинулась в яснах? A 20-year-old patient turned to the doctor with complaints of bleeding and pain in the gums that appeared 2 days after taking sulfadimezin. Objectively: gums the papillae and the gingival margin are hyperemic, swollen, and bleed with slight irritation. What pathology has developed in the gums?

Гострий катаральний гінгівіт Acute catarrhal gingivitis

Геморагічний гінгівіт Hemorrhagic gingivitis

Гострий виразковий гінгівіт Acute ulcerative gingivitis

Епулід Epulid

Хронічний катаральний гінгівіт Chronic catarrhal gingivitis

176 / 200
У хворого 48-ми років з обширним інфарктом міокарда розвинулась серцева недостатність. Який патогенетичний механізм сприяв розвитку серцевої недостатності у хворої? A 48-year-old patient with extensive myocardial infarction developed heart failure. What pathogenetic mechanism contributed to the development of heart failure in the patient?

Гостра тампонада серця Acute cardiac tamponade

Перевантаження об’ємом Volume overload

Перевантаження тиском Pressure overload

Реперфузійне ураження міокарда Myocardial reperfusion injury

Зменшення маси функціонуючих міо-кардіоцитів Reduction in mass of functioning myo-cardiocytes

177 / 200
До приймального відділення надійшов хворий з важкою щелепно-лицевою травмою. Який препарат йому необхідно ввести для зняття больового шоку? A patient with a severe maxillofacial injury was admitted to the reception department. What drug should be administered to him to relieve pain shock?

Мідокалм Midocalm

Пантогам Pantogam

Сиднокарб Sydnokarb

Ібупрофен Ibuprofen

Промедол Promedol

178 / 200
У померлого внаслідок серцевої недостатності на шкірі відзначаються сліди висипу у вигляді плям і крапок. В області криж, остистих відростків хребців - пролежні. При мікроскопічному дослідженні ЦНС: в судинах мікроциркуляторного русла і дрібних артеріях - деструктивно-проліферативний ендотромбоваскуліт з наявністю гранульом Попова, в серці - інтерстиційний міокардит. Який з перелічених діагнозів найбільш імовірний? The deceased due to heart failure has traces of a rash in the form of spots and dots on the skin. In the area of the sacrum, spinous processes of the vertebrae - bedsores. On microscopic examination of the central nervous system: in the vessels in the microcirculatory channel and small arteries - destructive-proliferative endothrombovasculitis with the presence of Popov's granuloma, in the heart - interstitial myocarditis. Which of the listed diagnoses is the most probable?

ВІЛ-інфекція HIV infection

Вузликовий періартеріїт Nodular periarteritis

Черевний тиф Typhoid

Ку-гарячка Coo fever

Висипний тиф Typhoid

179 / 200
У хворого під час гіпертонічного кризу виник геморагічний інсульт, внаслідок чого спостерігається відсутність довільних рухів, підвищення сухожильних рефлексів та тонусу м’язів лівих руки та ноги. Як називається таке порушення рухової функції? The patient had a hemorrhagic stroke during a hypertensive crisis, as a result of which there was a lack of voluntary movements, increased tendon reflexes and muscle tone of the left arm and leg. What is this called violation of motor function?

Млявий параліч Lethargic paralysis

Параплегія Paraplegia

Геміплегія Hemiplegia

Тетраплегія Tetraplegia

Моноплегія Monoplegia

180 / 200
В лабораторії вивчали вірулентність збудника дифтерії. При цьому проводили внутрішньоочеревинне зараження лабораторних тварин. В ході експерименту встановлена доза бактерій, яка викликає загибель 95% тварин. Яку одиницю вимірювання вірулентності визначали в лабораторії? In the laboratory, the virulence of the causative agent of diphtheria was studied. At the same time, laboratory animals were infected intraperitoneally. During the experiment, a dose of bacteria was established that causes the death of 95% of the animals. What is the unit of measurement of virulence determined in the laboratory?

LD50 LD50

DCL DCL

ІД ID

LD5 LD5

DLM DLM

181 / 200
При електронній мікроскопії у цитоплазмі клітини, поблизу ядра, виявлена мембранна органела, яка складається з 5-10 пласких цистерн, з розширеними периферичними ділянками, від яких від’єднуються маленькі пухирці - лізосоми. Назвіть цю органелу: Under electron microscopy, a membrane organelle consisting of 5-10 flat cisterns, with expanded peripheral areas, from which they detach small vesicles - lysosomes. Name this organelle:

Клітинний центр Cell Center

Цитоскелет Cytoskeleton

Рибосома Ribosome

Комплекс Гольджі Golgi Complex

Мітохондрія Mitochondria

182 / 200
При обробці фрезою великого кутнього зуба стоматолог інструментом, що зірвався, глибоко поранив щоку, пошкодивши при цьому не лише слизову оболонку але й м’яз. Який? While processing a large canine tooth with a milling cutter, the dentist deeply injured the cheek with a broken tool, damaging not only the mucous membrane but also the muscle. Which one?

Щічний Buccal

Жувальний Chewing

Щелепно-під’язиковий м’яз Maxillohyoid muscle

Коловий м’яз рота Orbital muscle

Великий виличний м’яз Major zygoma muscle

183 / 200
Шкіра людини дуже міцна на розрив. Відомо, що шкіра складається з епітеліальної тканини і двох видів сполучної тканини. Яка з нижче перерахованих тканин забезпечує міцність шкіри? Human skin is very tear-resistant. It is known that the skin consists of epithelial tissue and two types of connective tissue. Which of the following tissues ensures the strength of the skin?

Щільна неоформлена сполучна Dense unformed conjunction

Пухка сполучна тканина Loose connective tissue

Багатошаровий плоский епітелій Multilayered squamous epithelium

Одношаровий епітелій Single-layer epithelium

Перехідний епітелій Transitional epithelium

184 / 200
У хворого на стоматологічному прийомі виник напад пароксизмальної тахікардії, у зв’язку з чим йому ввели лідокаїн. З яким механізмом дії лідокаїну пов’язаний його протиаритмічний ефект? The patient had an attack of paroxysmal tachycardia during a dental appointment, in connection with which he was administered lidocaine. What mechanism of action of lidocaine is associated with its antiarrhythmic effect?

Потенціювання Potentiation

Мембранно-іонний Membrane ion

Сумація Summation

Кумуляція Cumulative

Антагонізм Antagonism

185 / 200
При огляді ротової порожнини стоматолог помітив появу у дитини перших постійних бічних різців. Який вік дитини? When examining the oral cavity, the dentist noticed the appearance of the child's first permanent lateral incisors. How old is the child?

7 років 7 years

1І років 1I years

9 років 9 years

13 років 13 years

6 років 6 years

186 / 200
Рибалка наловив риби із річки, трошки підсмажив її на вогнищі і з’їв, майже напівсиру. Через декілька тижнів потому в нього з’явились ознаки ураження печінки і підшлункової залози. Лабораторний аналіз фекалій показав наявність дрібних яєць гельмінта. Яким трематодозом найбільш імовірно заразився рибалка? A fisherman caught a fish from the river, fried it a little on the fire and ate it, almost half-raw. A few weeks later, he developed signs of damage to the liver and pancreas glands. Laboratory analysis of feces showed the presence of small helminth eggs. What trematode infection most likely infected the fisherman?

Опісторхоз Opistorchosis

Дикроцеліоз Dicroceliosis

Фасціольоз Fasciolosis

Шистосомоз Schistosomiasis

Парагонімоз Paragonimosis

187 / 200
Вивчається мітотичний поділ клітин епітелію ротової порожнини. Встановлено, що в клітині диплоїдний набір хромосом. Кожна хромосома складається з двох максимально спіралізованих хроматид. Хромосоми розташовані у площині екватору клітини. Ця картина характерна для такої стадії мітозу: The mitotic division of the cells of the oral cavity epithelium is being studied. It was established that the cell has a diploid set of chromosomes. Each chromosome consists of two maximally spiraled chromatids. The chromosomes are located in the plane of the cell's equator. This picture is typical for this stage of mitosis:

Прометафаза Prometaphase

Анафаза Anaphase

Метафаза Metaphase

Тілофаза Tylophase

Профаза Prophase

188 / 200
При огляді порожнини рота стоматолог виявив каріозну порожнину на поверхні коронки нижнього ікла, зверненої до першого малого кутнього зуба. Назвіть цю поверхню. When examining the oral cavity, the dentist found a carious cavity on the surface of the crown of the lower canine facing the first small canine tooth. Name this surface.

Facies occlusalis Facies occlusalis

Facies lingualis Facies lingualis

Facies distalis Facies distalis

Facies mesialis Facies mesialis

Facies vestibularis Facies vestibularis

189 / 200
В умовах експерименту проведено блокаду язико-глоткового нерву. При цьому буде спостерігатися зниження сприйняття такого подразника: In the conditions of the experiment, a blockade of the glossopharyngeal nerve was performed. At the same time, there will be a decrease in the perception of such a stimulus:

- -

Кисле Sour

Солодке Sweet

Гірке Bitter

Солоне Salty

190 / 200
Під час спортивних змагань боксер отримав сильний удар у живіт, що привело до нокауту через короткочасне падіння артеріального тиску. Які фізіологічні механізми викликали цей стан? During a sports competition, a boxer received a strong blow to the stomach, which led to a knockout due to a short-term drop in blood pressure. What physiological mechanisms caused this condition?

Подразнення симпатичних нервів Irritation of sympathetic nerves

Ішемія центральної нервової системи Ischemia of the central nervous system

Подразнення парасимпатичних нервів Irritation of parasympathetic nerves

Раптова зміна кількості рідини у організмі Sudden change in the amount of fluid in the body

Зміна транскапілярного обміну Transcapillary exchange change

191 / 200
У 12-річної дитини непереносимість ряду харчових продуктів. Їх вживання викликає алергічну реакцію у вигляді висипань на шкірі, що сверблять. Який антигістамінних засіб слід призначити, щоб не заважати шкільним заняттям дитини? A 12-year-old child has an intolerance to a number of food products. Their use causes an allergic reaction in the form of itchy rashes on the skin. What antihistamine should be prescribed so as not to interfere the child's schoolwork?

Лоратадин Loratadine

Ефедрин Ephedrine

Еуфілін Euphilin

Димедрол Diphenhydramine

Диклофенак Diclofenac

192 / 200
Чоловік 25-ти років звернувся до лікаря зі скаргами на біль в нижній щелепі справа, припухлість, високу температуру, озноб. При обстеженні макроскопічно відмічається відшарування окістя з накопиченням запального ексудату між ним і кісткою, з перифокальним набряком м’яких тканин та частково розплавленим окістям. Що розвинулося у хворого? A 25-year-old man went to the doctor with complaints of pain in the lower jaw on the right, swelling, high temperature, chills. During the examination, macroscopically, flaking of the periosteum with accumulation of inflammatory exudate between it and the bone, with perifocal edema of soft tissues and partially melted periosteum. What developed in the patient?

Гнійний періостит Suppurative periostitis

Локальний пародонтит Local periodontitis

Гранулюючий періодонтит Granulating periodontitis

Гангренозний пульпіт Gangrenous pulpitis

Катаральний гінгівіт Catarrhal gingivitis

193 / 200
У хворого з алкогольним ураженням печінки порушені процеси біотрансформації ксенобіотиків та ендогенних токсичних сполук. Зниження активності якого хромопротеїну може бути причиною цього? In a patient with alcoholic liver damage, the processes of biotransformation of xenobiotics and endogenous toxic compounds are disturbed. A decrease in the activity of which chromoprotein can be the cause of this?

Цитохром с1 Cytochrome c1

Цитохром Р-450 Cytochrome P-450

Цитохром b Cytochrome b

Цитохромоксидаза Cytochrome oxidase

Гемоглобін Hemoglobin

194 / 200
Піддослідному собаці ввели гормон, що призвело до збільшення швидкості клубочкової фільтрації за рахунок розширення приносної артеріоли і зменшення реабсорбції іонів натрію і води в канальцях нефрона. Який гормон було введено? A test dog was injected with a hormone, which led to an increase in the rate of glomerular filtration due to the expansion of the supply arteriole and a decrease in the reabsorption of sodium ions and water in the tubules of the nephron. What hormone was injected?

Тироксин Thyroxine

Окситоцин Oxytocin

Передсердний натрійуретичний Atrial natriuretic

Тестостерон Testosterone

Адреналін Adrenaline

195 / 200
Тривале лікування гіпофункції щитоподібної залози спричинило загальну дистрофію, карієс зубів, тахікардію, тремор кінцівок. Який засіб викликав побічні ефекти? Long-term treatment of hypothyroidism caused general dystrophy, dental caries, tachycardia, limb tremors. Which drug caused side effects?

Тирокальцитонін Tyrocalcitonin

Хумулін Humulin

Преднізолон Prednisone

Паратиреоїдин Parathyroidin

L-тироксин L-thyroxine

196 / 200
У жінки встановлено діагноз - рак шийки матки. З яким вірусом може бути асоційована ця патологія? A woman has been diagnosed with cervical cancer. What virus can this pathology be associated with?

Вірус простого герпесу тип 2 Herpes simplex virus type 2

Varicella-Zoster вірус Varicella-Zoster virus

Цитомегаловірус Cytomegalovirus

Аренавірус Arenavirus

Папілома вірус Papilloma virus

197 / 200
У пацієнта 59-ти років, що знаходиться на обстеженні в стаціонарі, виявлено глюкозурію, рівень глюкози в крові 3,0 ммоль/л. Найбільш імовірною причиною глюкозурії може бути: A 59-year-old patient undergoing examination in a hospital was diagnosed with glucosuria, the blood glucose level was 3.0 mmol/l. The most likely cause of glucosuria can be to be:

Нецукровий діабет Diabetes insipidus

Гіпертонічна хвороба Hypertensive disease

Пелагра Pellagra

Захворювання нирок Kidney disease

Мікседема Myxedema

198 / 200
При обстеженні підлітка, що страждає на ксантоматоз, виявлено сімейну гіперхолестеролемію Концентрація яких транспортних форм ліпідів підвищується при цьому захворюванні? During examination of a teenager suffering from xanthomatosis, familial hypercholesterolemia was revealed. The concentration of which transport forms of lipids increases in this disease?

Хіломікрони Chylomicrons

Ліпопротеїди високої щільності High-density lipoprotein

Ліпопротеїди дуже низької щільності Very low density lipoprotein

- -

Ліпопротеїди низької щільності Low-density lipoprotein

199 / 200
У молодої жінки, яка увійшла до приміщення з високою концентрацією тютюнового диму, раптово виникли рефлекторні кашель та спазм бронхів. Подразнення яких рецепторів викликало дані захисні рефлекси? A young woman who entered a room with a high concentration of tobacco smoke suddenly developed a reflex cough and spasm of the bronchi. Irritation of which receptors caused these protective reflexes?

Центральні хеморецептори Central chemoreceptors

Механорецептори легень Lung mechanoreceptors

Рецептори плеври Pleural receptors

Ірритантні рецептори Irritant receptors

Юкстамедулярні рецептори Juxtamedullary receptors

200 / 200
У хворого після тривалого психоемоційного перенапруження спостерігається підвищення артеріального тиску, що супроводжується серцебиттям, кардіалгіями, головним болем, запамороченням. Який процес відіграє домінуючу роль у формуванні артеріальної гіпертензії у даному випадку? After prolonged psycho-emotional overstrain, the patient has an increase in blood pressure, which is accompanied by palpitations, cardiac pain, headache, dizziness. What process plays a dominant role in the formation of arterial hypertension in this case?

Підвищення тонусу артеріол Increased tone of arterioles

Збільшення об’єму циркулюючої крові Increase in circulating blood volume

Підвищення частоти серцевих скорочень Increased heart rate

Збільшення серцевого викиду Increase in cardiac output

Підвищення тонусу венул Increased tone of venules